Download as pdf or txt
Download as pdf or txt
You are on page 1of 90

 

1  
 
   
 
 
   
 
 
 
 
 
 
 
 
 
 
 
 
 
By  Medical  Students    
 
For  Medical  Students  
 
 
 
 
 
 
 
 
 
 
 
 
 
 
 
 
 
 
 
 
 
 
 
 
 
 
 
 
2014  
 
“This  is  not  intended  to  be  your  reviewer.  Your  best  reviewer  is  of  Barbara  Bates,    
your  best  handout  is  your  ear,  and  your  best  teacher  is  yourself”    
 
 
  2  
 
 
CLINICAL  NEUROLOGY  by  Dra  Rosales                     3  
 
HEAD,  EYES,  EARS,  NOSE,  THROAT  by  Dra  Solis                   8  
 
EXTREMITIES  by  Dr  Paulino                       34  
 
CHEST  and  LUNGS,  BREAST  by  Dra  Lee                     56  
 
CARDIOLOGY  by  Dra  Deduyo                     65  
 
ABDOMEN  by  Dra  Cortez                         76  
 
 
 
 
 
 
 
Please  be  guided  that  answers  are  coming  from  medical  students;  do  not  rely.    
 
AKO  NA  MISMO  NAGSASABING  MALI  MALI  UNG  IBANG  MGA  SAGOT  DITO  KASI  AYAW  MAKINIG  NG  IBA  NA  NAG  EENCODE  
SABING  WAG  PALITAN  UNG  SAGOT.    
 
  -­‐  N  
 
 
 
 
 
 
 
 
 
 
 
 
 
 
 
 
 
 
 
 
 
 
 
 
 
 
 
 
 
  3  
NEUROLOGY  
1. Gait  ataxia  and  hypotonia  will  be  seen  in  a  patient  with  a  lesion  in  the  
a. Rostral  vermis   b.Posterior  lobe          c.Caudal  vermis   d.All       e.  A  and  C  
2. Manifestation  of  vermian  lesion  
a. Dysarthria   b.  Scanning  speech   c.  Nystagmus   d.All   e.A  and  c  
3. Manifestation  of  cerebellar  dysfunction  
a. Hypotonia   b.  Decomposition  of  movement     c.  Mild  aesthenia    d.All      
e.  A  and  C  
4. Test  for  arm  dystaxia  
a. Wrist  tapping  test   b.  Arm  pulling  test   c.  Thigh  patting  test     d.All     e.A  and  C  
5. True  in  a  patient  with  cerebellar  hemisphere  infarct  in  the  right  
a. Nystagmus   b.  Limb  ataxia     C.  Dysmetria  L     d.  All     e.  A  and  B  
6. Superficial  sensation  routinely  examined  
a. Romberg     b.  Asteriognosis     c.  Joint  position     d.  Pain                e.  C  and  D  
7. Manifestation  of  polyneuropathy  
a. Symetrical  distal  weakness     b.  Areflexia   c.  Preferential  sensory  loss  in  proximal  limb    
  d.  A  and  B       e.  All  
8. From  medial  to  lateral  (CTLS)  segmented  arrangement  of  fibers  in  the  spinal  cord  is  seen  in  the    
a. Fascicular  cuneatus/  gracilis   b.  Lateral  cortico  spinal       c.  Spinothalamic  
d.  All   e.  B  and  C  
9. TRUE  during  testing  of  sensory  function,    
a. Usually  done  with  patient’s  eyes  closed  
b. Should  compare  cornified  vs.  Non  cornified  areas  
c. Should  examine  symmetrical  dermatomal  area  
d. All  
e. A  and  C  
10. Pins  and  needles  sensation  
a. Dysaesthesia   b.  Paresthesia   c.  Alodynia     e.  Analgesia  
11. Biceps  reflex  is  subserved    
a. C2  –  c3     b.C4-­‐c5     c.  C5-­‐c6    d.  C7-­‐c8  
12. Variations  of  babinski  
a. Snout  reflex   c.  Hoffman’s   d.  Chaddock’s     e.All  
b. B  and  C  
13. Examination  of  motor  function  
a. Look  for  involuntary  movements   c. Inspect  muscle  size  
b. Look  for  coordinated  performance  of   d. all  
motor  acts  
14. TRUE  of  spastic  gate:  
a. Narrow  base  of  support  
b. Foot  plantar  flexed  and  everted  
c. Leg  externally  rotated  at  the  hip  
d. All  
e. A  and  C  
15. Upper  motor  neuron  paralysis:  
a. Muscles  affected  in  groups   e. A  and  C  
b. Rigidity  
c. Babinski  
d. All  
  4  

16. Fundoscopy   D       a.  Cranial  nerve  from  medulla  


17. Nystagmus   C       b.  Cranial  nerve  from  the  pons  
18. Say  EGG  A       c.  Cranial  nerve  from  the  midbrain  
19. Open  mouth   B       d.  Cranial  nerve  from  the  supra  tentorial  
20. Corneal  blink  reflex     B  

 
 
 
 
 
 
 
 
 
 
 
 
 
 
 
 
 
 
 
 
 
 
 
 
 
 
 
 
 
 
 
 
 
 
 
 
 
 
 
 
 
 
 
 
 
 
  5  

A. MATCHING  TYPE:  
1. A  stoke  patient  can  open  his  eyes,  moans  and  flexes  on  painful  stimulation  has  a  Glasgow  coma  scale  of:  
A. 3/15   B. 6/15   C. 10/15   D. 12/15  
2. The  main  objective  of  doing  deep  tendon  reflex  is  to:  
A. Differentiate  whether  the  lesion  is  UMN  or  LMN  
B. Differentiate  whether  the  lesion  is  brain  stem  or  spinal  cord  
C. Differentiate  whether  the  lesion  is  anterior  horn  cell  or  peripheral  nerve  
D. All  of  the  above  
3. Babinski  is  not  seen  in:  
A. Frontal  lobe  tumor   B. Brainstem  stroke   C. Spinal  cord   D. Diabetic  
compression   neuropathy  
4. A  high  stepped,  slapping  gate  is  usually  secondary  to:  
A. Parkinson’s  disease     B. Posterior  column   C. UMN  Lesion   D. Gait  apraxia  
lesion  
5. Paralysis  of  upward  gaze  is  usually  secondary  to:  
A. Optic  nerve  lesion   B. Optic  chiasm  lesion   C. Pineal  lesion   D. Pituitary  Lesion  
6. Not  a  feature  of  metabolic  encephalopathy  
A. Pupils  are  equal  and  reactive   C. Common  occurrence  of  movement  abnormality  
B. Severe  mental  status  alteration   D. Presence  of  lateralizing  sign  
7. Fatigue  with  exercise  is  usually  seen  in:  
A. Neuropathy   B. Myopathy   C. Neuromuscular  function  disorder   D. UMN  lesion  
8. A  unilateral,  dilated,  non  –  reactive  pupil  in  a  comatose  pate  will  indicate:    
A. Metabolic  encephalopathy   C. Pontine  hemorrhage  
B. Temporal  bone  herniation   D. Drug  overdose  
9. A  patient  complaining  of  weakness  that  he  can  only  move  his  extremities  against  gravity  is  graded  as:  
A. 1/5   B. 2/5   C. 3/5   D. 4/5  
10. Mental  status  examination  is  a  test  for  the  integrity  of:  
A. Cerebral  cortex   B. Cerebellum   C. Brainstem   D. Cranial  nerves  
11. An  optic  chiasm  lesion  would  cause:  
A. Failure  of   B. Paralysis  of  upward   C. Bitemporal  Hemianopsia   D. All  of  the  above  
convergence   gaze  
12. Lesion  in  the  cavernous  sinus  would  involve  the  following  cranial  nerves,  except:  
A. CN  III   B. CN  IV   C. CN  V   D. CN  VI  
13. Weakness  of  knee  extension  is  caused  by  a  lesion  of:  
A. Femoral  nerve   B. Peroneal  nerve   C. Sciatic  Nerve   D. Popliteal  nerve  
14. Features  of  upper  motor  neuron,  except:  
A. Spastic   B. (+)  Babinski   C. Atrophy   D. None  of  the  above  
15. Foot  drop  is  secondary  to  a  lesion  in  the:  
A. Femoral  nerve   B. Peroneal  nerve   C. Lateral  cutaneous   D.  
nerve  
16. The  first  cranial  nerve  to  be  affected  by  increased  intracranial  pressure  is:  
A. CN  III   B. CN  IV   C. CN  VI   D. CN  VI  
17. Signs  of  peripheral  nerve  lesion,  except:  
A. Atrophy   B. Fasciculations   C. Spasticity   D. Hyporeflexia  
18. Neurologic  examination  in  a  comatose  patient  includes  the  following,  except:  
A. Fundoscopy   B. Pupillary  light  reflex   C. Mental  status   D. Cerebellar  examination  
examination  
19. Signs  and  symptoms  of  myopathy  includes  the  following,  except:  
A. Proximal  weakness   B. Atrophy   C. Hyperreflexia   D. None  of  the  above  
20. The  most  reliable  sign  of  UMN  lesion:  
A. Hyperreflexia   B. Atrophy  and  fasciculation   C. (+)  Babinski   D. Spasticity  
21. Presence  of  cheery  red  spots  seen  in  the  retina  by  fundoscopy  is  seen  in:  
A. Tay  Sach’s  disease   C. Inreacrania  hemorrhage  
  6  

B. Subarachnoid  hemorrhage   D. Von  Recklinghausen’s  disease  


22. Presence  of  café  au  lait  spots  seen  in  fundoscopy  is  secondary  to  
A. Tay  Sach’s  disease   C. Inreacrania  hemorrhage  
B. Subarachnoid  hemorrhage   D. Von  Recklinghausen’s  disease  
23. Ataxia  is  seen  in  the  following  lesions,  except:  
A. Dorsal  column   B. Cerebellum   C. Spinocerebellar  tract   D. None  of  the  above  
24. Movement  disorder  associated  with  basal  ganglia  lesions,  except:  
A. Dysmetria   B. dystonia   C. Bradykinesia   D. Tremor  
 
 
 
25. In  the  lesion  of  the  superior  orbital  fissure  the  following  are  true,  except:    
A. Weakness  of  the  lateral  movement  of  the  eye  
B. Weakness  of  the  medial  movement  of  the  eye  
C. Weakness  of  the  superior  and  inferior  movement  of  the  eye  
D. None  of  the  above  
 
B. ASSOCIATION  TYPE  
A  –  If  1,2,3  are  correct   D  –  Only  4  is  correct  
B  –  If  1  and  3  are  correct   E  –  if  all  are  correct  
C  –  If  2  and  4  are  correct  
 
E  26.  Neurologic  examination  is  mandatory  if  the  patient  complains  of:  
1. Loss  of  consciousness   2. Weakness   3. Urinary  incontinence   4. Dementia  
E    27.Sensory  examination  includes  the  following:  
1. Crude  touch   2. Position   3. Vibration   4. Temperature  
B    28.  Bitemporal  hemianopsia  is  secondary  to:  
1. Pituitary  adenoma   2. Brainstem  lesion   3. Uncal  herniation   4. Metabolic  encepalopathy  
E    29.  The  appearance  of  the  following  reflexes  would  indicate  frontal  lobe  lesion    
1. Palmomental   2. Sucking   3. Grasp   4. Babinski  
A    30.  Babinski  could  be  present  if  you  have  lesions  of  the:  
1. Supratentorial   2. Posterior  fossa   3. Spinal   4. Anterior  horn  cell  
A    31.  Neurologic  examination  in  a  comatose  patient  includes:  
1. Light  reflex   2. Fundoscopy   3. Mental  status   4. Cerebellar  exam  
C    32.  Retinal  hemorrhage  could  be  seen  in:  
1. Optic  neuritis   2. Malignant   3. Cerebral  infarction   4. Raptured  aneurysm  
hypertension  
B      33.  Mental  status  examination  includes:  
1. Level  of  consciousness     2. Memory   3. Mood   4. Position  
B      34.  Fundoscopy  is  valuable  if  you  are  entering:  
1. Increased  intracranial  pressure   2. Demyelinating  disorder   3. Raptures  aneurysm   4. Cerebral  infarction  
C      35.  Signs  and  symptoms  of  myopathy  
1. Absent  sensory  changes   2. Proximal  weakness   3. Atriphy   4. Hyporeflexia  
A      36.  Movement  disorders  associated  with  basal  ganglia  lesion  
1. Athetosis   2. Ballismus   3. Parkinson’s  disease   4. Dystonia  
C      37.  Small,  pinpoint  pupils  are  associated  with:  
1. Amphetamines   2. Opiate  overdose   3. Uncal  herniation   4. Pontine  lesion  
overdose  
D      38.  Large,  dilated  pupils  are  associated  with:  
1. Cocaine  use   2. Heroine  use   3. Opiate  overdose   4. Amphetamine  overdose  
E      39.  Horner’s  syndrome:  
1. Mitotic  pupils   2. Ptosis   3. Facial  anhydrosis   4. Dilated  pupils  
  7  

D      40.  Melkensson  syndrome  


1. Painful  edema   2. Caused  by  herpes  zoster  virus   3. Pitting  edema   4. Scrotal  tongue  
 
C. MATCHING  TYPE  
I   41. Phonation   A. Cranial  I  
G   42. Numbness  of  the  face   B. Cranial  II  
B   43. Visual  acuity   C. Cranial  III  
E   44. Clenching  of  teeth   D. Cranial  IV  
K   45. Shoulder  shrug   E. Cranial  V  
L   46. Tongue  atrophy   F. Cranial  VI  
C   47. Convergence   G. Cranial  VII  
    H. Cranial  VIII  
    I. Cranial  IX  
    J. Cranial  X  
    K. Cranial  XI  
    L. Cranial  XII  
 
B   48. CN  II  –  III   A. Corneal  reflex  
A   49. CN  V  –  VII   B. Pupillary  light  reflex  
C   50. CN   I X   –   X   C. Gag  reflex  
 
 
 
 
 
 
 
 
 
 
 
 
 
 
 
 
 
 
 
 
 
 
 
 
 
 
 
 
 
 
  8  

HEENT  
 
1.  Unilateral  headache  that  can  be  localized  behind  the  eyes  –  CLUSTER  
2.  Enlarged  blind  spot  occurs  in  a  condition  affecting  the  optic  nerve  
3.  Rhinoscopy  (ndi  dapat  tamaan)  –  NASAL  SEPTUM  
4.  polyps  –    MEDIAL  MEATUS  
5.  Family  history  –  MIGRAINE  
6.  Hyperthyroidism  –  GRAVE’S  DISEASE  
7.  button  like  –CHANCRE  IN  SYPHILIS  
8.  maplike  –GEOGRAPHICAL  
9.  caused  by  deficiency  in  riboflavin  and  niacin  –  SMOOTH  TONGUE  
10.  ear  pull  (adults)  –  UP  &  BACK  
11.  unilateral  painless  –  RETINAL  WALL  
12.  examining  the  oropharynx  use  a  tongue  depressor  –  DISTAL  HALF  OF  TONGUE  
13.  white  optic  disc  and  tiny  vessel  are  absent  –  OPTIC  ATROPHY  
14.  fissured  tongue  –  APPEARED  WITH  INCREASING  AGE  
15.  nutritional  deficiency  (cold  sore)  –  ANGULAR  CHEILITIS  
16.  caused  by  trauma  –  SUBCONJUNCTIVAL  HEMORRHAGE  
17.  tonsilar  LN  pulsation    
  a.  carotid  artery  
  b.  ext.  Jugular  Vein  
  c.  lymph  adenopathy  
  d.  bruit  
 
18.  convergence  test  –  

19.  Tug  test  painful    in  –  OTITIS  EXTERNA  

ADDITIONAL:  
 
• Otitis  Externa  –  pale,  moist  ,  narrow  
• Retracted  eardrum  
• Soft  nodule  in  the  thyroid  gland  –  GRAVE’s  
• Behind  the  eye  –  Cluster  
• Riboflavin  def.  and  chemotx.  –  smooth  
• Exposure  to  sumlight  –  lip  carcinoma  
• Factor  to  carcinoma  –  Actinic  Cheilitis  
• Worsen  in  noisy  environmanet  –  sensorineural  hearing  loss(SNHL)  
• Presbyopic  –  better  seen  further  away  
Central    loss  
• Button  like  infection  –  Angular  cheilitis  
• Ill  fitting  dentures  
• Fissure  tongue  –  increasing  age  
• Sign  of  lip  canar?  –  Actinic  cheilitis  
  9  

 
1.  Hold  target  at  the  midline  and  at  eye  level  gradually  moving  the  target  toward  the  bridge  of  the  nose.  
  a.  Lid  lag  
  b.  Convergence  
  c.  Accommodation  
  d.  Confrontation  
 
2.  For  the  test  above  
  a.  This  is  normally  maintained  at  a  distance  of  2-­‐3  inches  from  the  nasal  bridge  of  the  nose  
  b.  Watch  for  the  appearance  of  white  sclera  between  the  iris  and  the  upper  lid.  
  c.  Usually  a  person  sees  both  sets  of  fingers  at  the  same  time.  
  d.  There  is  pupillary  constriction  in  the  opposite  eye  
 
3.  There  is  poor  convergence  in  
  a.  Hypothyroidism  
  b.  Hypertension  
  c.  Hyperthyroidism  
  d.  Diabetes  mellitus  
 
4.  Absent  red  reflex  suggest  
  a.  Normal  eye  
  b.  Artificial  eye  
  c.  Papilledema  
  d.  Hyperthyroidism  
 
5.  External  auditory  canal  is  often  swollen  ,  narrowed  moist,  pale  and  tender.  It  may  be  reddened.  This  is  
  a.  Chronic  otitis  externa  
  b.  Acute  otitis  externa  
  c.  Acute  purulent  otitis  media  
  d.  acute  otitis  media  
 
6.  This  is  not  a  special  eye  technique  for  eye  examination  
  a.  Nasolacrimal  duct  obstruction  
  b.  Inspection  of  the  upper  palpebral  conjunctiva  
  c.  For  assessing  protruding  eyes  
  d.  Extraocular  muscle  test  
7.    Inspection  of  the  nasal  cavity  through  the  anterior  naris  us  usually  limited  to  the  following:  
  a.  Vestibule  
  b.  Superior  turbinates  
  c.  Sphenoid  sinus  
  d.  Frontal  sinus  
8.  Spinning  sensation  is  accompanied  by  nystagmus  and  ataxia  
  a.  Vertigo  
  b.  Dizziness  
  c.  Tinnitus  
  d.  Disequilibrium  
9.  Retracted  tympanic  membrane  
  a.  More  conical  
  b.  Seen  on  Acute  suppurative  otitis  media  
  c.  Loss  of  bony  landmarks  
  d.  Accentuated  bony  landmarks  
 
10.  Local  cause  of  nosebleeding  
  10  

  a.  Flying  
  b.  Hypertension  
  c.  Nose  Picking  
  d.  Leukemia  
  e.  English  
 
11.  Deacrease  facial  mobility  and  characteristic  stare  (Parkinson’s  disease)  
12.  Head  is  elongated  with  bony  prominence  of  the  forehead,  nose  and  lower  jaw  (Acromegaly)  
13.  swelling  usually  appears  first  in  the  eyes  and  in  the  morning  (Nephrotic  syndrome)  
14.  hair  is  dry,  coarse  and  sparse  with  periorbital  edema.  Lateral  eyebrows  thin  (Myxedema)  
15.  Red  cheeks,  hirsutism  and  “moonface”  (Cushing’s  syndrome)  
16.  May  accompany  lipid  disorders  (Xanthelasma)  
17.  Tearing  is  prominent.  Nasolacrimal  duct  obstruction  is  also  noted  (Dacryocyctitis)  
18.  Usually  points  inside  the  lid  rather  that  the  lid  margin  (Chalazion)  
19.  A  painful,  tender  red  infection  in  a  gland  at  the  margin  of  the  eyelids  (Sty)  
20.  Drooping  of  eyelids  (Ptosis)  
 
Q:  Unilateral  headache  that  can  be  localized  behind  the  eye  
A.  Cluster  
 
Q:  Enlarge  Blind  spot  occurs  in  a  condition  affecting  optic  nerve  
A:    
 
Q:  Rhinoscopy  (hindi  dapat  tamaan)  
A:  Nasal  septum  
 
Q:  Polyps  
A:  Medial  Meatus  
 
Q:  Family  History  
A:  Migraine  
 
Q:  Hyperthyroidism  
A:  Grave’s  disease  
 
Q:  Button  like  
A:  Chancre  syphilis  
 
Q:  Map-­‐like  
A:  Geographical  
 
Q:  Caused  by  a  deficiency  in  riboflavin  and  Niacin  
A:  smooth  Tongue  
 
Q:  Ear  pull  (adult)  
A:  Up  and  back  
 
Q:  Unilateral,  painless  
A:  Retinal  Wall  
 
Q:  Examining  oropharynx,  use  tongue  depressor:  
A:  Distal  half  of  tongue  
 
  11  

Q:  White  optic  disc  and  tiny  vessel  are  absent  


A:  Optic  Atropy  
 
Q:  Fissured  tongue  
A:  Appeared  with  increasing  age  
 
 
 
• Subarachnoid  Hemorrhage  -­‐  nausea  and  vomiting,  possible  loss  of  consciousness,  neck  pain  

 
• Ptosis  -­‐  cause  myasthenia  gravis,  damage  to  the  oculomotor  nerve  (CN  III),  damage  to  the  sympathetic  nerve  
supply  (  Horner's  syndrome)  

• Ectropion  -­‐  the  margin  of  the  lower  lid  is  turned  outward,  exposing  the  palpebral  conjunctiva  

• Inflammation  of  the  Lacrimal  Sac  (Dacryocystitis)  -­‐  swelling  between  the  lower  eyelid  and  the  nose  

• Horner's  Syndrome-­‐  small  affected  pupil,  reacts  briskly  to  light  and  near  effort,    ptosis  present,  loss  of  
sweating  on  forehead,  heterochromia  

• Argyll-­‐Robertson  pupils  -­‐  small,  irregular  pupils  that  accomodate  but  do  not  react  to  light  indicate  CNS  syphilis  

• Normal  arterial  wall  is  transparent;  Normal  light  reflex  is  narrow  

• Silver  wiring  -­‐  occasionally  a  portion  of  a  narrowed  artery  develops  such  as  an  opaque  wall  that  has  no  blood  
is  visible  within  it.  

• Microaneurysms  -­‐  tiny,  round  red  spots  seen  commonly  but  not  exclusively  in  and  around  the  macular  area;  
minute  dilatations  of  very  small  retinal  vessels,  but  the  vascular  connections  are  too  small  to  be  seen  
opthalmoscopically  

• Neovascularizations  -­‐  formation  of  new  blood  vessels;  more  numerous,  more  tortuous,  and  narrower  than  
other  blood  vessels  in  the  area  and  form  disorderly  looking  red  arcades  

• Hypertensive  Retinopathy  -­‐  marked  arteriolar  venous  crossing  changes  are  seen,  copper  wiring  of  the  
arterioles  is  present.  Cotton  wool  spot  is  seen  just  superior  to  the  disc.  

• Nonproliferative  Retinopathy  (  Moderately  severe)  -­‐  tiny  red  dots/  microaneurysms  

• Proliferative  Retinopathy  (  Neovascularization)  -­‐  new  preretinal  vessels  arising  on  the  disc  extening  across  the  
disc  margins.  Visual  acuity  is  still  normal,  but  risk  for  visual  loss  is  high  

 
• Keloid  -­‐  a  firm,  nodular,  hypertrophic  mass  of  scar  tissue  (binding)  extending  beyond  the  area  of  injury  

• Tophi  -­‐  deposit  of  uric  acid  crystals  characteristic  of  chronic  tophaceous  gout  

• Cutaneous  cyst/  Sebaceous  cyst  -­‐  a  dome  shaped  lump  in  the  dermis  forms  a  benign  closed  firm  sac  
attached  to  the  dermis  

• Rheumatoid  Nodules  -­‐  small  lump  on  the  helix/antihelix  and  additional  nodules  elsewhere  on  the  hands  
along  the  surface  of  the  ulna  distal  to  the  elbow  
  12  

• Acute  Otitis  Media  with  Purulent  Effusion  -­‐  caused  by  bacterial  infection  earache,  fever  and  hearing  loss.  
Hearing  loss  is  of  the  conductive  type  

• Sensorineural  loss  -­‐  weber's  test:  sound  lateralizes  to  good  ear  

 
• Angular  Cheilitis  -­‐  softening  of  the  skin  at  the  angles  of  the  mouth,  fissuring  

• Chancre  of  Syphilis  -­‐  appear  on  the  lip,  firm  button-­‐like  lesion  

 
• Large  Normal  Tonsils  -­‐  normal  tonsils  may  be  enlarged;  protrude  medially  beyond  the  pillars  and  even  to  
the  midline  

• Diptheria  -­‐  dull  red,  gray  exudate  (pseudomembrane)  is  present  on  the  uvula,  pharynx  and  tongue  

• Koplik's  spots  -­‐  early  sign  of  measles,  small  white  specks  that  resembles  grains  of  salt  

• Acute  Necrotizing  Ulcerative  Gingivitis  -­‐  ulcers  develop  in  the  interdental  papilla  

• Hutchinson's  teeth  -­‐  sides  of  these  teeth  show  normal  contours;  sides,  shaping  of  the  teeth  are  unaffected  

• Smooth  tongue  (Atrophic  Glossitis)  -­‐  lost  its  papillae,  deficiency  in  riboflavin,  niacin,  folic  acid,  Vit.  B12,  
pyridoxine,  iron  

• Apthous  ulcer  (Canker  sores)  -­‐  painful,  round/oval  ulcer  that  is  white/yellowish  gray  and  surrounded  by  a  
halo  of  reddened  mucosa  

• Diffuse  Enlargement  -­‐  endemic  goiter  

• Hyperthyroidism  -­‐  tachycardia  

 
1. Enlarged  skull  may  signify:  Hydrocephalus  or  Paget’s  disease  of  Bone  
2. 20/200  vision  meaning:  at  20  ft.,  the  patient  can  read  print  that  a  person  with  normal  vision  could  read  at  
200  feet.  
3. Absence  of  a  red  reflex:  Cataract  (opacity  of  lens),  detached  retina,  retinoblastoma  
4. Light  rays  from  a  distance  focus  on  the  anterior  of  retina:  Myopia  
Light  rays  from  a  distance  focus  on  the  posterior  of  retina:  Hyperopia  
5. Loss  of  venous  pulsation  in  pathologic  conditions  like  head  trauma,  meningitis,  or  mass  lesions  may  be  an  
early  sign  of:  Elevated  ICP  
6. Canal  is  swollen,  narrowed,  moist,  pale,  tender,  reddened:  Acute  Otitis  externa  
7. Unilateral  Conductive  hearing  loss:  Sound  is  heard  in  the  Impaired  ear  
8. Unilateral  Sensorineural  Hearing  loss:  Sound  is  heard  in  the  good  ear  
9. Conductive  hearing  loss:  BC  >  AC  
10. Mucosa  is  reddened  and  swollen:  Viral  rhinitis  
11. Mucosa  is  pale,  bluish,  or  red:  Allergic  rhinitis  
12. Submental  Lymph  node  (maytanong  bout  sa  Submental  LN)  
13. Basic  landmark  for  palpating  Thyroid  gland:  Thyroid  cartilage  and  Cricoid  cartilage??  
14. Nausea,  Vomiting:  Migraine/Subarachnoid  
15. Sudden  movements  of  the  head  may  be  associated  with:  Brain  tumor  
16. Spinning  sensation:  Vertigo  
17. Left  Homonymous  Hemianopsia:  Right  Optic  Radiation  
18. Damage  to  Oculomotor  nerve:  Ptosis  
19. Eye  no  longer  drains  satisfactorily:  Ectropion  
  13  

20. Bilateral  Exopthalmos:  Grave’s,  hyperthyroidism  


Unilateral  exophthalmos:  Grave’s  dse/  Tumor/  Inflammation  of  the  orbit  
21. Painful,  Tender,  Red  infection  of  the  margin  of  the  eyelid:  Stye  
22. Pupil  is  large,  regular  and  usually  unilateral;  reaction  to  light  is  severely  reduced/  slowed  or  absent:  Adie’s  
pupil  (Tonic  pupil)  
23. Loss  of  venous  pulsation;  disc  vessels  more  visible,  more  numerous:  papilledema  
24. Arteries  show  areas  of  focal/generalized  narrowing:  hypertension  
25. Arteries  become  full  and  somewhat  tortuous;  Inc  light  reflex:  Copper  wiring  
26. Presence  of  cotton-­‐wool  spot:  Hypertensive  Retinopathy  
27. May  tanong  on  page  267,  di  ko  maalala  pero  meron  
28. Softening  of  the  skin  at  the  angles  of  the  mouth  followed  by  fissuring:  Angular  Cheilitis  
29. Firm  lesion  on  the  lip:  Chancre  of  Syphilis  
30. Reddened  throat  without  exudate:  Pharyngitis  
31. Smooth  tongue  that  has  lost  its  papillae:  Atrophic  glossitis  
32. Headache  is  severe  and  sudden  onset:  Subarachnoid  hemorrhage/  Acute  meningitis  
33. Sudden  unilateral  visual  loss  is  pinless:  Retinal  detachment/  retinal  vein  occlusion/  central  retinal  artery  
occlusion/  vitreous  hemorrhage/  macular  degeneration  
*If  visual  loss  is  painful:  corneal  ulcer/  uveitis/  acute  glaucoma/  optic  neuritis  
34. Bilateral  and  painless  visual  loss:  d/t  cholinergics,  anticholinergics  and  steroids/  Chemical,  radiation  
exposure  
35. People  having  trouble  understanding  speech;  noisy  environment  makes  hearing  worse:  Sensorineural  loss  
36. Local  cause  of  epistaxis:  Trauma  (nose  picking)  –  most  common  
37. Enlarged  blind  spot:  Glaucoma/  Optic  neuritis/  Papilledema  
 
 
HEENT  
1. In  primary  position,  R  eye    deviates  laterally  but  cannot  move  medially  
a. R  lateral  rectus  palsy  
b. R  oblique  rectus  palsy  
c. R  medial  rectus  palsy  
d. R  superior  rectus  palsy  
 
2. Renal  artery  of  HTN-­‐  focal  narrowing    
 
3. A  portion  of  a  narrowed  artery  develops  such  an  opaque  wall  that  no  blood    is  visible  with  in  it  
a. silver  artery  or  silver  wire  artery  
b. copper  wire  
c. normal  artery  
d. retinal  sclerosis  
 
4. most  important  attribute  for  head  ache.  
 A.  Chronologic  pattern    
B.  Quality    
C.  Location    
D.  Timing  
 
5. Holding  a  pencil  and  moving  toward  the  bridge  of  the  nose  (ganyan  hung  thought  nung  question,  mahaba  
kasi  yung  question  e)  Ans:  Convergence  test  
 
  14  

6. True  about  the  test  above  


a. 5-­‐8  cm  distance  from  the  nose….  
b. …  
c. …  
d. …  
 
7.  
8.  
9.  
10.  
11.  
12. Inspection  of  anterior  nares  is  limited  to:  
a. Vestibule  
b. Sphenoid  sinus  
c. Sup.  Turbinate  
d. Frontal  sinus  
 
13. Spinning  sensation  with  nystagmus  and  ataxia    
a.  Vertigo  
 b.  Dizziness  
 c.  Tinnitus    
d.  Dysequilibrium    
ans.  A.  VERTIGO?    
 
14. Retracted  tympanic  membrane….  
 
15. Local  cause  of  nose  bleeding    
A.  Flying    
B.Hypertension    
C.  Nose  picking  
D.  Leukemia  
 
16. In  opthalmoscopic  examination,-­‐The  view  is  limited  to  posterior    structure  
17.  
18. Benign  lesion  associated  with  antibiotic  therapy-­‐  hairy  tongue  
19. Examination  of  LN  is  done  by-­‐  palpation  
20. Basic  landmark  of  thyroid  gland-­‐  Cricothyroid  

 
 
 
 
 
 
 
 
 
  15  

Test  I:  Choose  the  BEST  answer  


1.  In  opthalmoscopic  Examination  
a.  The  view  is  limited  to  posterior  structure  
b.  Pheripheral  structures  can  be  evaluated  in  the  absence  of  mydriatic  drops  
c.  (+)3  or  (+)4  diopters  will  allow  you  to  see  the  anterior  structures  clearly  
d.  Red-­‐orange  reflex  is  normally  not  visualize  first  
 
2.  A  (-­‐)  lens  is  used  in  
a.  Hyperopic  eyes  
b.  Aphakic  eyes  
c.  Myopic  eye  
d.  Astigmatism  
 
3.  Physical  sign  of  retrosternal  goiter  
a.  Venous  engorgement  
b.  Tender  thyroid  
c.  Thyroid  bruit  
d.  Regional  lymphnode  enlargement  
 
4.  Examinatin  of  the  lymphnode  is  primarily  by  
a.  Inspection  
b.  Palpation  
c.  Auscultation  
d.  X-­‐ray  
 
5.  Basic  landmark  for  thyroid  gland  examination  
a.  Cricoid  cartilage  
b.  Thyroid  cartilage  
c.  Trachea  
d.  Sternocleidomastoid  
 
6.  Primary  lesion  from  posterior  2/3  of  the  scalp  and  nasopharynx  
a.  Submental  lymphnodes  
b.  Posterior  cervical  triangle  
c.  Anterior  crvical  triangle  
d.  Supraclavicular  lymphnode  
 
7.  Nasal  flaring  is  associated  with  
a.  Respiratory  distress  
b.  Chronic  nasal  obstruction  
  16  

c.  Mouth  breathing  
d.  Allergic  rhinitis  
 
8.  A  hole  in  nasal  septum  is  commonly  caused  by  
a.  Syphilis  
b.  Tuberculosis  
c.  Cocaine  abuse  
d.  Repeated  trauma  in  picking  off  crusts  
 
9.  Headache  presents  on  aakening  
a.  Migraine  
b.  Brain  tumor  headache  
c.  Meningitis  
d.  Subarachnoid  hemorrhage  
 
10.  Sudden  unilateral  painless  visual  loss  
a.  Acute  glaucoma  
b.  Corneal  ulcer  
c.  Uveitis  
d.  Retinal  vein  occlusion  
 
Test  II:  Match  the  abnormalities  of  the  lips  with  description  below  
a.  Angular  cheilitis  
b.  Cold  sore  
c.  Chancre  of  syphilis  
d.  Carcinoma  of  the  lips  
e.  Angioedema  
 
   B    11.  Produce  recurrent  and  painful  eruptions  of  the  lips  and  surrounding  skin  
   A      12.  Maybe  due  to  ill-­‐fitting  dentures  
   C      13.  Highly  infectious,  firm,  button-­‐like  lesion  that  ulcerates  and  may  become  crusted  
     D      14.  Fair  skin  and  prolonged  exposure  to  the  sun  are  common  risks  factors  
   A      15.  It  may  be  due  to  nutritional  deficiency  
 
Test  III:  Match  the  pattern  of  hearing  loss  with  description  below  
a.  Conductive  hearing  loss  
b.  Sensorineural  hearing  loss  
 
   A      16.  One  cause  is  otitis  media  
   B      17.  In  weber  test,  sound  lateralizes  to  good  ear  
  17  

   B      18.  In  Rinne  test,  normal  pattern  prevails  


   A      19.  Voice  maybe  loud  because  hearing  is  difficult  
   A      20.  Usual  ageof  onset  childhood  and  adulthood,  up  to  age  40  
 
 
 
 
 
 
 
 
 
 
 
 
 
 
 
 
 
 
 
 
 
 
 
 
 
 
 
 
 
 
 
 
 
 
 
 
 
  18  

HEENT  
a.  Conductive  hearing  loss  
b.  Sensorineural  hearing  loss  
 
1. Voice  may  be  loud  because  the  patient  has  trouble  hearing  his  or  her  own  voice        B      
2. Age  of  onset  is  most  often  in  childhood  and  young  adulthood,  up  to  age  40        A      
3. Otitis  media        A      
4. Sound  lateral  to  good  ear  in  Weber  test        B      
5. AC>BC,  Rinne  test        B      
 
6. Basic  landmark  –  cricoid  cartilage  
7. Lesion  in  the  posterior  2/3  of  the  scalp  and  nasopharynx  –  subscapular  
8. Retrosternal  goiter  –  venous  engorgement  
9. Examination  of  the  lymphnode  –  palpation  
10. Ophalmoscopic  examination  
a.  The  view  is  limited  to  posterior  structure  
b.  Pheripheral  structures  can  be  evaluated  in  the  absence  of  mydriatic  drops  
c.  (+)3  or  (+)4  diopters  will  allow  you  to  see  the  anterior  structures  clearly  
d.  Red-­‐orange  reflex  is  normally  not  visualize  first  
11. A  (-­‐)  lens  is  used  in  
a.  Hyperopic  eyes  
b.  Aphakic  eyes  
c.  Myopic  eye  
d.  Astigmatism  
 
12. Firm,  button-­‐like  lesion  –  chancre  of  syphilis  
13. Risk  factors  are  fair  skin,  and  prolonged  exposure  to  sun  –  carcinoma  of  the  lip  
14. Nutritional  insufficiency  –  angular  cheilitis  
15. Ill-­‐fitting  dentures  –  angular  cheilitis  
16. Painful  vesicular  lesions  in  angle  of  the  mouth  –  herpes  simplex/cold  sore  
17. Unilateral,  painless  visual  loss  
a.  Acute  glaucoma  
b.  uveitis  
c.  retinal  vein  occlusion  
d.  all  of  the  above  
18. Headache  upon  waking  up  –  migraine  
19. Nasal  flaring  associated  to  
a.  Respiratory  distress  
b.  Chronic  nasal  obstruction  
c.  Mouth  breathing  
  19  

d.  Allergic  rhinitis  
20. Hole  in  the  basal  septum  most  common  in  
a.  Syphilis  
b.  Tuberculosis  
c.  Cocaine  abuse  
d.  Repeated  trauma  in  picking  off  crusts  
 
 
 
 
 
 
 
 
 
 
 
 
 
 
 
 
 
 
 
 
 
 
 
 
 
 
 
 
 
 
 
 
 
 
  20  

Test  I:  Choose  the  BEST  answer  


1.  In  opthalmoscopic  Examination  
a.  The  view  is  limited  to  posterior  structure  
b.  Pheripheral  structures  can  be  evaluated  in  the  absence  of  mydriatic  drops  
c.  (+)3  or  (+)4  diopters  will  allow  you  to  see  the  anterior  structures  clearly  
d.  Red-­‐orange  reflex  is  normally  not  visualize  first  
 
2.  A  (-­‐)  lens  is  used  in  
a.  Hyperopic  eyes  
b.  Aphakic  eyes  
c.  Myopic  eye  
d.  Astigmatism  
 
3.  Physical  sign  of  retrosternal  goiter  
a.  Venous  engorgement  
b.  Tender  thyroid  
c.  Thyroid  bruit  
d.  Regional  lymphnode  enlargement  
 
4.  Examinatin  of  the  lymphnode  is  primarily  by  
a.  Inspection  
b.  Palpation  
c.  Auscultation  
d.  X-­‐ray  
 
5.  Basic  landmark  for  thyroid  gland  examination  
a.  Cricoid  cartilage  
b.  Thyroid  cartilage  
c.  Trachea  
d.  Sternocleidomastoid  
 
6.  Primary  lesion  from  posterior  2/3  of  the  scalp  and  nasopharynx  
a.  Submental  lymphnodes  
b.  Posterior  cervical  triangle  
c.  Anterior  crvical  triangle  
d.  Supraclavicular  lymphnode  
 
 
 
 
  21  

7.  Nasal  flaring  is  associated  with  


a.  Respiratory  distress  
b.  Chronic  nasal  obstruction  
c.  Mouth  breathing  
d.  Allergic  rhinitis  
 
8.  A  hole  in  nasal  septum  is  commonly  caused  by  
a.  Syphilis  
b.  Tuberculosis  
c.  Cocaine  abuse  
d.  Repeated  trauma  in  picking  off  crusts  
 
9.  Headache  presents  on  aakening  
a.  Migraine  
b.  Brain  tumor  headache  
c.  Meningitis  
d.  Subarachnoid  hemorrhage  
 
10.  Sudden  unilateral  painless  visual  loss  
a.  Acute  glaucoma  
b.  Corneal  ulcer  
c.  Uveitis  
d.  Retinal  vein  occlusion  
 
Test  II:  Match  the  abnormalities  of  the  lips  with  description  below  
a.  Angular  cheilitis  
b.  Cold  sore  
c.  Chancre  of  syphilis  
d.  Carcinoma  of  the  lips  
e.  Angioedema  
 
   B    11.  Produce  recurrent  and  painful  eruptions  of  the  lips  and  surrounding  skin  
   A      12.  Maybe  due  to  ill-­‐fitting  dentures  
   C      13.  Highly  infectious,  firm,  button-­‐like  lesion  that  ulcerates  and  may  become  crusted  
     D      14.  Fair  skin  and  prolonged  exposure  to  the  sun  are  common  risks  factors  
   A      15.  It  may  be  due  to  nutritional  deficiency  
 
 
 
 
  22  

Test  III:  Match  the  pattern  of  hearing  loss  with  description  below  
a.  Conductive  hearing  loss  
b.  Sensorineural  hearing  loss  
 
   A      16.  One  cause  is  otitis  media  
   B      17.  In  weber  test,  sound  lateralizes  to  good  ear  
   B      18.  In  Rinne  test,  normal  pattern  prevails  
   A      19.  Voice  maybe  loud  because  hearing  is  difficult  
   A      20.  Usual  ageof  onset  childhood  and  adulthood,  up  to  age  40  
 
 
 
 
 
 
 
 
 
 
 
 
 
 
 
 
 
 
 
 
 
 
 
 
 
 
 
 
 
 
 
  23  

HEENT  
a.  Conductive  hearing  loss  
b.  Sensorineural  hearing  loss  
 
1. Voice  may  be  loud  because  the  patient  has  trouble  hearing  his  or  her  own  voice        B      
2. Age  of  onset  is  most  often  in  childhood  and  young  adulthood,  up  to  age  40        A      
3. Otitis  media        A      
4. Sound  lateral  to  good  ear  in  Weber  test        B      
5. AC>BC,  Rinne  test        B      
 
6. Basic  landmark  –  cricoid  cartilage  
7. Lesion  in  the  posterior  2/3  of  the  scalp  and  nasopharynx  –  subscapular  
8. Retrosternal  goiter  –  venous  engorgement  
9. Examination  of  the  lymphnode  –  palpation  
10. Ophalmoscopic  examination  
a.  The  view  is  limited  to  posterior  structure  
b.  Pheripheral  structures  can  be  evaluated  in  the  absence  of  mydriatic  drops  
c.  (+)3  or  (+)4  diopters  will  allow  you  to  see  the  anterior  structures  clearly  
d.  Red-­‐orange  reflex  is  normally  not  visualize  first  
11. A  (-­‐)  lens  is  used  in  
a.  Hyperopic  eyes  
b.  Aphakic  eyes  
c.  Myopic  eye  
d.  Astigmatism  
 
12. Firm,  button-­‐like  lesion  –  chancre  of  syphilis  
13. Risk  factors  are  fair  skin,  and  prolonged  exposure  to  sun  –  carcinoma  of  the  lip  
14. Nutritional  insufficiency  –  angular  cheilitis  
15. Ill-­‐fitting  dentures  –  angular  cheilitis  
16. Painful  vesicular  lesions  in  angle  of  the  mouth  –  herpes  simplex/cold  sore  
17. Unilateral,  painless  visual  loss  
a.  Acute  glaucoma  
b.  uveitis  
c.  retinal  vein  occlusion  
d.  all  of  the  above  
18. Headache  upon  waking  up  –  migraine  
19. Nasal  flaring  associated  to  
a.  Respiratory  distress  
b.  Chronic  nasal  obstruction  
c.  Mouth  breathing  
  24  

d.  Allergic  rhinitis  
20. Hole  in  the  basal  septum  most  common  in  
a.  Syphilis  
b.  Tuberculosis  
c.  Cocaine  abuse  
d.  Repeated  trauma  in  picking  off  crusts  
 
 
 
 
 
 
 
 
 
 
 
 
 
 
 
 
 
 
 
 
 
 
 
 
 
 
 
 
 
 
 
 
 
 
  25  

Our  Lady  of  Fatima  University  


College  of  Medicine  –  Regular  class  
nd
2  semester  2013-­‐2014  
Clinical  Medicine  
HEENT  
Name:  _____________________________________  Section:  ______________  Date:  _________  Score:______  
 
1.  Major  attributes  to  headache  
                           a.  Location   c.  Chronological  pattern  
  b.  Quality  
2.  Headache  is  episodic  and  tends  to  peak  several  hours.  This  is:  
  a.  Migraine     c.  brain  tumor  
  b.  Cluster  headache   d.  meningitis  
3.  Nausea  and  vomiting  is  common  in:  
  a.  Meningtis     c.  Brain  tumor  
  b.  Tension  headache   d.  Rebound  
4.  Cough,  sneezing  ang  changing  patternof  the  head  can  increase  the  pain  from:  
  a.  tension  headache   c.  Brain  tumor  
  Meningitis/migraine   d.  cluster  headache  
5.  Aging  vision:   Presbyopia  
6.  Bilateral  bilateral  painful  eye  maybe  due  to  the  following  
  a.  Chronic  radiation  exposure   c.  Central  retinal  occlusion  
  b.  Cholinergic  medication     d.  corneal  ulcer  
7.  Horizontal  diplopia  
  a.  weakness  or  paralysis  of  EOM     c.  lesion  CN  III  or  CN  V  
  b.  Lesion  CN  III  or  IV       d.  problems  in  cornea  
8.  In  conductive  hearing  loss  
  a.  Have  particular  trouble  understanding  speech  
  b.  Noisy  environment  makes  hearing  worse  
  c.  Problem  in  inner  ear  
  d.  noisy  environment  help  
9.  perception  that  the  patient  or  the  environment  is  rotating  or  spinning.  This  is:  
  a.  Tinnitus  
  b.  Vertigo  
  c.  Dizziness  
  d.  Meniere’s  disease  
10.  perceived  sound  that  has  no  external  stimulus:   a.  dizziness  
11.  sensation  of  spinning:   Vertigo  
12.  Fever,  pharyngeal  exudates,  anterior  lymphadenopathy  without  cough:  
  a.  viral  pharyngitis     c.  Diptheria  
  b.  Strep  Pharyngitis     d.  Infectious  Mononucleosis    
13.  Hyperthyroidism  
  a.  Cold  intolerance       c.  Weight  loss  
  b.  Preference  of  warm  clothing       d.  Decreased  sweating  
nd
14.  Cause  of  blindness  in  African  American  and  2  leading  cause  of  blindness  overall?  
a.  Cataract     c.  weight  loss  
  b.  Glaucoma       d.  decreased  sweating  
15.  20/40  corrected  is:    
  a.  The  patient  can  read  the  line  40  with  glasses  
  b.  Patient  can  read  the  line  without  glasses  
  c.  Vision  is  normal  
  d.  Patient  is  presbyopic  
 
  26  

16.  An  enlarged  blind  spot  occurs  in  condition  affecting  optic  nerve  such  as:  
  a.  Optic  atrophy     c.  Glaucoma  
  b.  Cataract     d.  Papilledema  
17.  Contraindication  for  mydriatic  drops  
  a.  Coma       c.  Arcus  senillis  
  b.  Cataract     d.  pterygium  
18.  The  Tug  test  is  painful  in:  
  a.  Otitis  media     c.  chronic  otitis  externa  
  b.  Acute  otitis  externa   d.  purulent  otitis  media  
19.  Unilateral  sensory  neural  loss  
  a.  Sound  is  heard  at  the  good  ear  
  b.  Sound  is  heard  in  the  impaired  ear    
  c.  Due  to  impact  cerumen  
  d.  due  to  eardrum  perforation  
20.  The  nasal  mucosa  is  pale,  bluis  or  red.  This  suggests:  
  a.  Viral  rhinitis     c.  allergic  rhinitis  
  b.  Acute  sinusitis     d.  normal  mucosa  
21.  A  triangular  thickening  of  the  bulbar  conjunctiva:    a.  pterygium  
22.  Characteristic  of  Optic  atrophy  
  a.  Visible  optic  vessel   c.  absent  optic  vessel  
  b.  tiny  optic  vessel   d.  no  vessel  pulsation  
23.  Normal  retinal  artery  
  a.  Arterial  wall  transplant       c.  Narrow  light  reflex  
  b.  narrow  column   of  blood  vessel     d.  focal  narrowing  
24.  Fullness  and  popping  sound  in  the  ear  with  mild  conductive  hearing  loss  and  ear  pain  
  a.  Acute  otitis  media     c.  with  effusion  
  b.  Chronic  otitis  externa  
25.  Bulging  eardrum  
  a.  Hearing  loss  is  sensorineural   c.  accentuated  
  b.  Obscured       d.  changes  in  atmospheric  pressure  
26.  The  patient  complains  of  earache  and  hearing  loss.  The  eardrum  are  reddened.  Losses  its  landmark  and  
buldges  laterally  towards  the  examiners  eye  
  a.  Acute  otitis  media  with  purulent  effusion  
  b.  Chronic  otitis  externa  
  c.  Chronic  otitis  media  
  d.  Acute  otitis  externa  
27.  The  skin  of  the  ear  canal  is  often  thickened,  red  and  itchy:  Chronic  otitis  externa  
28.  Button  like  lesion:   Chancre  syphilis  
29.  Benign  condition  that  may  follow  antibiotic  therapy:  
  a.  Geographic  tongue   c.  smooth  tongue  
  b.  Fissured  tongue   d.  hairy  tongue  
30.  Deficiency  in  riboflavin,  niacin,  folicacid,  B12  and  pyrodoxin  
  a.  Atrophic     c.  Hairy  
  b.  Fissured     d.  Geographic  
31.  Basic  landmark  of  the  thyroid  
  a.  Thyroid  cartilage   c.  Trachea  
  b.  Cricoid  cartilage   d.  Isthmus  
32.  Furosemide  –  medication  that  affect  the  hearing  
33.  Nose  picking  for  local  cause  of  epistaxis  
34.  Rhinitis  medicamentosa  –  excessive  use  of  decongestant  
35.  Cause  of  excessive  tearing  
36.  Absence  of  red  reflex  
37.  Bilateral/unilateral  exopthalmus  
  27  

38.  Actinic  cheilitis  


39.  Diphtheria  
40  Canker  sore  –  a  painful  round  or  ovalulcer  that  is  white  or  yellowish  gray  that  is  surrounded  by  halo  reddened  
mucosa  
41.  Tophi  –  deposits  of  uric  acid  crystals  
 
 
 
 
 
 
 
 
 
 
 
 
 
 
 
 
 
 
 
 
 
 
 
 
 
 
 
 
 
 
 
 
 
 
 
 
 
 
 
 
 
 
 
 
 
 
 
  28  

 
1. Vision  of  20/200  means  that    
a. at  20  ft,  the  patient  can  read  print  that  a  person  with  normal  vision  could  read  at  200  ft  
b. at  200  ft,  the  patient  can  read  print  that  a  person  with  normal  vision  could  read  at  20  ft.  
c. the  larger  the  first  number,  the  worse  the  vision  
d. normal  vision  
2. An  image  from  the  upper  nasal  visual  field  strikes  the  
a. Upper  temporal  area   c. Lower  temporal  area  
b. Lower  nasal  area   d. Upper  nasal  area  
3.  Sees  better  when  the  card  is  farther  away  
a. Nearsightednesss   c. Presbyopia  
b. Myopia   d. hyperopia  
4.  Absence  of  red  reflex  indicates  
a. Artificial  eye   c.    
b. Opacity  of  lens   d.    
5.  An  enlarged  blind  spot  occurs  in  
a. Grave’s   c. Optic  neuritis  
b. Retroorbital  tumor   d. Diabetic  neuropathy  
6.  Headache  from  errors  of  refraction  include  
a. nearsightedness   c. astigmatism  
b. farsightedness   d. myopia  
7. Testing  near  reaction  is  used  in  diagnosis  of  
a. Argyll  Robertson  pupil   c. Oculomotor  nerve  paralysis  
b. Anisocoria   d. Horner’s  syndrome    
8.  Fixed  defects  (scotoma)  are  seen  in  
a. Retina   c. Lens  
b. Cornea   d. Pupil    
9.  Excessive  tearing  from  increased  production  is  due  to  
a. Corneal  irritation   c. Entropion  
b. Extropion   d. Nasolacrimal  duct  obstruction  
10.  Most  important  attribute  of  headache  
a. Severity   c. Quality  
b. Chronological  pattern   d. location  
11. Hyperthyroidism  
a. Intolerance  to  cold   c. Involuntary  weight  loss  
b. Preference  for  warm  clothing   d. Decreased  sweating    
12. The  tug  test  is  painful  in  
a. Otitis  media   c. Chronic  otitis  externa  
b. Acute  otitis  externa   d. Purulent  otitis  media    
13.  Bilateral,  painless  change  in  refractory  may  be  due  to  
a. Chemical  exposure   c. Steroids  
b. Radiation  exposure   d. Diabetes  
14.  Family  history  may  be  positive  is  
a. Tension   c. Migraine  
b. Cluster   d. Medication  obveruse  
15.  The  eardrum  itself  is  scarred,  no  landmarks  visible,  often  closes  in  healing  processs  
a. Tympanosclerosis   c. Otosclerosis  
b. Perforation  of  eardrum   d. Serous  effusion  
 
Retinal  Arteries  and  Arteriovenous  Crossing  
 
C   16.  Arteries  may  show  focal  or  generalized  narrowing     a.  Copper  wore  
B   17.  Opaque  wall  and  no  blood  visible         b.  Silver  wire  
  29  

A   18.  Arteries  close  to  the  disc  become  full  and  somewhat  torturous   c.  Retinal  Arteries  in  Hypertension  
C   19.  Arterial  wall  is  invisible  A-­‐V  crossing  is  visible       d.  Normal  Retinal  Artery  
C   20.  Light  reflex  is  narrow  about  one-­‐fourth  the  diameter  of  blood  column  
 
Pupillary  Abnormalities  
 
D   21.  pupils  that  accommodate  but  do  not  react  to  light       a.  Anisocoria  
C   22.  dialted  pupil  is  fixed  to  light  and  near  effort       b.  Adie’s  pupil  
B   23.  slow  accommodation  causes  blurred  vision       c.  Oculomotor  nerve  paralysis  
B   24.pupil  is  large  regular,  usually  unilateral         d.  Argyll  Robertson  pupil  
A   25.causes  include  blunt  trauma  to  the  eyes,  open-­‐angle  glaucoma  
 
 
Diplopia  
 
A   26.  Images  are  side  by  side         a.  Horizontal  diplopia  
A   27.  Caused  by  palsy  of  CN  III  or  IV       b.  Vertical  diplopia  
D   28.  Diplopia  in  one  eye  with  the  other  closed       c.  Both    
B   29.  Images  are  on  top  of  each  other         d.  None  
A   30.  One  kind  of  this  diplopia  is  physiologic  
 
  30  

 
Hearing  Loss  
 
A   31.  Abnormality  is  usually  visible  except  in  the  otosclerosis       a.  Conductive  hearing  loss  
B   32.  In  Weber  test,  room  noise  not  appreciated       b.  Sensorinueral  hearing  loss  
B   33.  Voice  is  loud  because  hearing  is  difficult         c.  Both  
C   34.  To  estimate  hearing,  test  one  ear  at  a  time       d.  None  
B   35.  In  Rinne  test,  the  normal  pattern  prevails  
 
Eardrum  
 
D   36.  Pink,  grayish,  intact           a.  Serous  Effusion  
C   37.  Caused  by  acute  purulent  infection  of  the  middle  ear   b.  Perforated  Eardrum  
B   38.  Eardrum  itself  is  scarred,  no  landmarks  are  visible   c.  Acute  Otits  Media  with  Purulent  Effusiion  
A   39.  Amber  fluidbehind  the  eardrum  is  characteristic.     d.  Normal  Eardrum  
           Air  bubbles  can  be  seen  with  the  amber  fluid  
C   40.  Eardrum  is  bulging  and  redden,  most  landmarks  are  obscured  
 
Clinical  Findings  
 
D   41.  Diptheria           a.  Benign  midline  lump  
F   42.  Apthous  ulcer           b.  Nasal  sputum  
G   43.  Epulis           c.  Poor  convergence  
E   44.  Fever  blister           d.  Pseudomembrane  
J   45.  Microaneurysm         e.  Cold  sore  
H   46.  Sentinel  node           f.  Canker  sore  
B   47.  Transilumination         g.  Pregnancy  tumor  
I   48.  Hyperthyroidism         h.  Thoracic  malignancy  
K   49.  Hypothyroidism         i.  (+)  lidlag  
A   50.  Torus  palatus           j.  Diabetes  
              k.  None  of  the  above  
 
 
1. Major  attributes  to  headache  
a. Location  
b. Quality  
c. Chronologic  pattern  
d. Hindi  kya  AOTA  to?  Kasi  tama  lahat  
2. Headache  is  episodic  and  tends  to  peak  after  several  hours.  
a. Migraine  
b. Cluster  
c. Brain  tumor  
d. Meningitis  
3. Nausea  and  vomiting  is  common  in  
(MIGRAINE  ang  sagot,  wala  s  choices.  Lahat  ng  choices  walang  assoc.  nausea  and  vomiting)  
a. Meningitis  
b. Tension  
c. Brain  tumor  
d. Rebound  
  31  

 
4. Cough,  sneezing,  changing  position  of  the  head  can  increase  the  pain  from:  
a. Tension  headache  –  sustained  muscle  tension  
b. Migraine  –  noise  and  bright  lights  
c. Brain  tumor  (and  sinusitis)  
d. Cluster  –  alcohol  
5. Aging  vision  –  PRESBYOPIA  
6. Bilateral  painful  eye  
a. Chronic  radiation  exposure  
b. Cholinergic  medication  –  bilateral,  painless  
c. Central  retinal  artery  occlusion  –  unilateral,  painless  
d. Corneal  ulcer-­‐  unilateral,  painful  
7. Horizontal  diplopia  
a. Weakness  or  paralysis  of  EOM  
b. Lesion  of  CN  III  or  IV  
c. Lesion  of  CN  III  or  VI  
d. Corneal  problems  
8. Conductive  hearing  loss  
a. Have  particular  trouble  understanding  speech  –  sensorineural  
b. Noisy  environment  makes  hearing  worse  –  sensorineural  
c. Problem  in  inner  ear  –  sensorineural  
d. Noisy  environment  may  help  
9. Perception  that  the  patient  or  the  environment  is  spinning\  
a. Tinnitus  –  perceived  sound  without  external  stimulus  
b. Vertigo  
c. Dizziness  
d. Meniere’s  disease  –  tinnitus  +  hearing  loss  +  vertigo  
10. Perceived  sound  without  external  stimulus  –  TINNITUS  
11. Sensation  of  spinning  –  VERTIGO  (hindi  kaya  dizziness  to?)  
12. Fever,  pharyngeal  exudates,  anterior  lymphadenopathy,  no  cough  
a. Viral  pharyngitis  
b. Strep  pharyngitis  
c. Diphtheria  
d. Infectious  mononucleosis  
13. Hyperthyroidism  
a. Cold  intolerance  –  hypo  
b. Preference  of  warm  clothing  –  hypo  
c. Weight  loss  
d. Decrease  sweating  –  hypo  
nd
14. Leading  cause  of  blindness  in  African  American  and  2  leading  cause  of  blindness  overall  
a. Cataract  
b. Glaucoma  
c. Macular  degeneration  
d. Retinal  detachment  

 
  32  

 
15. 20/40  corrected  is  
a. Patient  can  read  line  40  with  glasses  
b. Patient  can  read  the  line  without  glasses  
c. Vision  is  normal  
d. Presbyopia  
16. An  enlarged  blind  spot  occurs  in  conditions  effecting  the  optic  nerve  such  as:  (glaucoma,  optic  neuritis,  
papilledema)  
a. Optic  atrophy  
b. Cataract  
c. Glaucoma  
d. Papilledema  
17. Contraindications  for  mydriatic  drops  (head  injury,  coma,  suspicion  of  narrow  angle  glaucoma)  
a. Coma  
b. Cataract  
c. Arcus  senilis  
d. Pterygium  
18. Tug  test  is  painful  in    
a. Otitis  media  
b. Acute  otitis  externa  
c. Chronic  otitis  externa  
d. Purulent  otitis  media  
19. Unilateral  sensorineural  loss  
a. Sound  is  heard  in  good  ear  
b. Sound  is  heard  in  impaired  ear  –  unilateral  conductive  
c. Impacted  cerumen  
d. Eardrum  perforation  
20. The  nasal  mucosa  is  pale,  bluish,  or  red  
a. Viral  rhinitis  
b. Acute  sinusitis  
c. Allergic  rhinitis  
d. Normal  mucosa  
21. Triangular  thickening  of  bulbar  conjunctiva  –  PTERYGIUM  
22. Characteristic  of  optic  atrophy  
a. Visible  optic  vessel  
b. Tiny  optic  vessel  
c. Absent  optic  vessel  
d. No  vascular  pulsation  
23. Normal  retinal  artery  
a. Arterial  wall  is  transparent  
b. Narrow  column  of  blood  vessel  –  HPN  
c. Narrow  light  reflex  
d. Focal  narrowing  –  HPN  
24. Fullness  and  popping  sound  in  the  ear  with  mild  conductive  hearing  loss  and  ear  pain  –  SEROUS  EFFUSION  
25. Bulging  eardrum  (Parang  wala  tamang  sagot.  Dapat  acute  otitis  media  with  purulent  effusion)  
a. Hearing  loss  is  sensorineural  –  conductive  
  33  

b. Obscured  
c. Accentuated  
d. Changes  in  atmospheric  pressure  –  serous  effusion  
26. The  patient  complains  of  earache  and  hearing  loss.  The  eardrums  are  reddened,  loses  its  landmark,  and  
bulges  lateral  towards  examiner’s  eye.  
a. acute  otitis  media  with  purulent  effusion  
b. chronic  otitis  media  externa  
c. chronic  otitis  media  
d. acute  otitis  externa  
27. skin  of  the  ear  canal  is  often  thickened,  red  and  itchy  –  CHRONIC  OTITIS  EXTERNA  
28. Button  like  lesion  –  CHANCRE  IN  SYPHILLIS  
29. Benign  condition  that  may  follow  antibiotic  therapy      
a. Geographic  tongue  –  dorsum  shows  scattered  smooth  areas  denuded  of  papillae  
b. Fissured  tongue  –  increasing  age  
c. Smooth  tongue  /  atrophic  glossitis  –  deficiency  of  riboflavin,  niacin,  folic  acid,  vitB12,  pyridoxine,  iron  
or  treatment  with  chemotherapy  
d. Hairy  tongue  –  HIV,  AIDS  
30. Deficiency  of  riboflavin,  niacin,  folic  acid,  B12,  pyridoxine  
a. Atrophic  glossitis  
b. Fissured  
c. Hairy  
d. Geographic  
31. Basic  landmark  of  thyroid  cartilage  
a. Thyroid  cartilage  
b. Cricoid  cartilage  
c. Trachea  
d. Isthmus    
 
 
 
 
 
 
 
 
 
 
 
 
 
 
 
 
 
 
 
  34  

EXTREMITIES  

Clinical  Medicine  EXTREMITIES  


 
1. Test  for  nerve  irritation..  
 
a. Trendelenberg  –  HIP  DISLOCATION  (PAINLESS  LIMPING)  
b. Faber  Test  
c. Thomas  Test  –  HUGH  OWEN  THOMAS  SIGN  (TEST  THIGH  AND  HIP  JOINT)  LUMBAR   LORDOSIS,  
PELVIC  DISEASE,  HIP  JOINT  DISEASE  
d. None  
 
THOMAS  SIGN-­‐  TEST  FLEXION  (FLATTENING  OF  LUMBAR  CURVE  IS  OBSERVE)  
 
2. Type  of  primary  joint  articulation  that  allows  free  movement..  
 
a. Fibrous  
b. Synovial  
c. Cartilagenous  
d. All  
 
3. Manifestation  /  Cause  of  extreme  dorsiflexion  foot  
 
a. Ruptured  Achilles  tendon  –  absence  plantar  flex  (simmond’s  test)  
b. Soleus  tear  –  malformation  leg  severe  pain  and  tenderness  
c. Gastrocnemius  strain  
d. All  
 
4. Site  of  muscle  of  the  rotator  except  
 
a. Infraspinatus  
b. Supraspinatus  
c. Teres  major  
d. None  
 
5. ROM  of  elbow  
Ans.  Flexion,  extension,  supination,  pronation  
 
6. Anterior  knee  cyst  –  INFLAMMATION  OF  THE  BURSA  
 
a. Housemaid  –  PREPATELLAR  BURSITIS  
b. Clergyman  –  INFRAPATELLAR  BURSITIS  
c. Baker’s  cyst  –  MORRANT  BAKER  POPLITEAL  ARTERY  ENTRAPMENT  
d. A  &b  
 
BAKER  
-­‐  COMPLICATION  OF  RA  
 
7. Osteoarthritis  
 
a.Heberden’s  nodes  
b.  Bouchard’s  nodes  
c.  asymmetric..  
  35  

d.  all  
 
8. Palmar  erythema  except  
 
a.portal  HPN  
b.aplastic  anemia  
c.  pregnancy  
d.valvular  heart  dse  
 
 
9. Wrist  ROM  
 
Ans.  Flexion,  extension,  abduction,  adduction  
 
10. True  regarding  neck  ROM  
 
a.flexion  and  extensionat  the  skull  and  C1  –  C3-­‐C7  
b.rotation  at  c1  and  c2  –  ATLANTO-­‐AXIAL  
c.a&b  
d.  none  
 
11. Visual  examination  from  behind..  landmarks  except  
 
a.spinous  process  of  T1  -­‐  PALPATION  
b.iliac  crest  
C.POSTERIOR  SUPERIOR  ILIAC  SPINE  
d.  dimple  of  venus  
 
12. Test  for  Lumbar  lordosis  
ANS.  THOMAS  TEST  
 
13. Examination  of  motor  function  
 
a.look  for  tremor  
b.look  for  coordinated  movement  
c.muscle  size  
d.all  
 
14. Pronated  hand  dropped  from  wrist  
a. Carpal  tunnel  syndrome  
b. Ulnar  nerve  palsy  
c. Median  nerve  palsy  (MEDIAN  =  RADIAL)  
d. None  
 
15. Lateral  deviation  from  midline  
ANS.  GENU  VALGUM  
 
 
 
 
16. Rheumatoid  arthritis  
 
A.MIP  &POP  INVOLVEMENT  
  36  

B.HAYGARTH’S  NODES  
c.Bouchard’s  nodes  -­‐  OA  
d.all  
e.a&b  
 
17. Unequal  shoulder  length  
 
A.SCOLIOSIS  
B.SPRENGEL’S  DEFORMITY  
c.a&b  
d.none  
 
18. Unequal  leg  length  except  
Ans.  Kyphosis  
 
19. Long  narrow  nails  except  
 
a.hypopotuitarism  
b.eunochoidism  
c.  cretinism  -­‐  SQUARE  
d.none  
 
20. Lordosis  except  
Ans.  A.  …convexity  
 
 
NOTE:  Kindly  Recheck  all  the  answers!  
 
God  bless!  
 
 
 
 
 
 
 
 
  37  
A. Matching  type    
     
E  1.  Drumstick  fingers   A.  Passive  hyperextension    
C  2.  Spider  fingers   B.  Radial  nerve  injury    
       3.  Subungal  haemorrhage   C.  Arachnodactyly    
A  4.  Graenlen's  test   D.  Ulnar  nerve  injury    
H  5.  Lesague's  test   E.  Clubbing    
E  6.  Hypertrophic  osteoarthropathy   F.  SBE    
G  7.  Trendelenburg's  sign   G.  Hip  dislocation    
       8.  Tinel'  sign   H.  Straight  leg  test    
K  9.  Infrapatellar  bursitis   I.  Carpal  tunnel  syndrome    
     10.  Genu  recurvatum   J.  Housemaid's  knee    
M  11.  Acromegaly   K.  Clergyman's  knee    
F  12.  Azure  half-­‐moon   L.  Small  patella    
D  13.  Hypothenar     M.  Square  &  round  nail  plate    
-­‐-­‐-­‐-­‐-­‐   N.  -­‐-­‐-­‐-­‐-­‐    
-­‐-­‐-­‐-­‐-­‐   O.  -­‐-­‐-­‐-­‐-­‐    
P.  -­‐-­‐-­‐-­‐-­‐    
 
 
 
 
 
B.  MULTIPLE  CHOICE  
 
16.  Thumb  in  Filipino  is:  
A.  Hinlalaki   C.  Hinlalato  
B.  Palasingsingan   D.  None  
 
17.  Hinlalato  is:  
A.  Thumb   C.  Middle  finger  
B.  Index  finger   D.  None  
 
18.  Severe  carpal  tunnel  syndrome  is  managed  by:  
A.  NSAIDs   C.  Complete  rest  of  the  wrist  
B.  Physical  therapy   D.  Surgical  decompression  
 
19.  Not  a  malposture  of  the  hand:  
A.  Claw  hand   C.  Wrist  drop  
B.  Ape  hand   D.  Polydactyly  
 
20.  Most  common  hypothesis  of  clubbing:  
A.  Unknown  mechanism   C.  Parrot  beak  nails  
B.  Hypoxia   D.  (?)  
 
21.  Lateral  bending  of  the  spine:  
A.  Atlanto-­‐axial  joint   C.  Atlanto-­‐occipital  
B.  Midcervical  vertebra   D.  C3-­‐C7  
 
22.  Eggshell  nails:  
A.  Vitamin  A  deficiency   C.  Thyrotoxicosis  
B.  Hypochromic  anemia   D.  Bronchiectasis
 
 
  38  

23.  Brittle  nails:  


A.  Onycholysis   C.  Onychocrytosis  
B.  Onychorrhexis   D.  Onychocryptosis  
 
24.  Ram's  horn  nails:  
A.  Onycholysis   C.  Onychocrytosis  
B.  Onychorrhexis   D.  Onychocryphosis  
 
25.  Fixation  of  the  2nd  toe  in  flexion:  
A.  Hallus  valgus   C.  Hammer  toe  
B.  Hallus  rigidus   D.  Callus  
 
26.  Point  of  reference  of  the  fingers:  
A.  Wrist  joint   C.  Ulnar  nerve  
B.  Middle  finger   D.  Radial  nerve  
 
27.  Rheumatoid  arthritis  is  mostly  seen  in  
A.  Fingers   C.  Elbow  
B.  Knees   D.  Hip  joint  
 
28.  Pain  referred  to  the  shoulder:  
A.  Pneumonia   C.  Winged  scapula  
B.  Supraspinatus  tendinitis   D.  All  
 
29.  Excessive  transverse  growth  of  the  nail  plate  causing  lateral  edge  to  lacerate:  
A.  Onychauxis   C.  Ram's  horn  nails  
B.  Onychocryptosis   D.  Onychogryphosis
 
 
30.  One  of  the  following  regarding  the  examination  of  the  spine  is  incorrect:  
A.  From  the  side  inspect  the  spinal  profile  
B.  From  behind  inspect  the  lateral  curve  
C.  Percuss  the  spine  with  a  neurological  hammer  
D.  Use  your  thumb  to  palpate  for  the  para  vertebral  area  
 
31.  One  of  the  following  is  a  cause  of  kyphosis  in  postmenopausal  women:  
A.  Osteoporosis   C.  Osteus  deformans  
B.  Faulty  posture   D.  Ankylosing  spondylitis  
 
32.  When  the  legs  deviate  towards  the  midline  and  the  knees  farther  apart,  it  is  called:    
A.  Genu  varum   C.  Genu  recurvatum  
B.  Genu  valgum   D.  None  of  the  above  
 
33.  Presence  of  blood  in  the  joint  cavity:  
A.  Hemarthrosis   C.  Hemoptysis  
B.  Hematemesis   D.  Hematochezia  
 
 
34.  The  length  of  the  lower  extremities  can  be  measured  by:  
A.  ASIS  to  the  tip  of  the  medial  malleolus  with  the  tape  crossing  the  patella  
B.  ASIS  to  the  tip  of  lateral  malleolus  
C.  From  iliac  crest  to  Achilles  tendon  
D.  From  iliac  crest  to  medial  malleolus  
  39  

 
35.  When  doing  the  lumbar  puncture,  iliac  crest  is  at:  
A.  4th  lumbar  vertebrae   C.  T12-­‐  L1  
B.  L1-­‐L2   D.  S4  
 
 
36.  Extreme  dorsiflexion  of  the  foot  
A.  Ruptured  Achilles  tendon   C.  Soleus  tear  
B.  Fracture  of  the  tibial  shaft   D.  Fracture  of  the  fibular  shaft  
 
37.  Pressure  diverticulum  of  the  synovial  sac  protruding  thru  the  joint  capsule  of  the  knee  
A.  Baker's  cyst   C.  Pyarthrosis  
B.  Popliteal  abscess   D.  Prepatellar  bursitis  
 
38.  Swelling  in  the  popliteal  fossa  
A.  Baker's  cyst   C.  Prepatellar  bursitis  
B.  Popliteal  abscess   D.  Infrapatellar  bursitis  
 
39.  Which  of  the  following  is  the  most  common  cause  of  painful  swelling  in  young  males?  
A.  Paget's  disease  
B.  Ankylosing  spondylitis  
C.  Pott's  disease  
D.  Osteoporosi
  40  
 
 
UPPER  EXT  
1.  Hands  (ROM  –  ABDUCTION,  ADDUCTION,  EXTENSION,  FLEXION)  
Condition  impaired  ROM  =  FAT  
1. Fibrosis  of  palmar  fascia  (DUPUYTREN’S  CONTRACTURE)  
2. Arthritis  
3. Tenosynovitis  (inflammation  of  tendon  sheaths)  
 
Size  hands  
LONG  
ACROMEGALY   AFTER  EPIPHYSIS  CLOSE  (ADULT)  
EXCESSIVE  PRODUCTION  GH  
NOT  SYMMETRICAL  
GIGANTISM   BEFORE  EPIPHYSIS  CLOSE  (CHILD)  
PROPORTIONATE  AND  SYMMETRIC  
ND
NOTE:  BOTH  ARE  2  TO  TUMOR  +  BY  AN  EXCESS  OF  SOMATOTROPHIC  HORMONE  FROM  EOSINOPHILIC  ADENOMA  OF  
ANTERIOR  PIT  GLAND  
 
LOND  AND  SLENDER  HANDS  
*SPIDER  FINGERS  
SMALL,  THICK  HANDS  
CRETINISM  (CHILD)   DWARF  PX  
MYXEDEMA  (OLD)   ABSOLUTE  DEF  OF  TH  
HANDS  SHORT,  THICK,  FAT  
RADIUS  MAY  BE  SHORTENED  
MONGOLISM   CHROMOSOMAL  ABN  
HANDS  SHORT,  THICK,  THUMB  DIVERGES  FROM  
NEARER  THE  WRIST  THAN  NORMAL  
LITTLE  FINGER  IS  CURVED  (RADIAL  WARD)  
 
MALPOSTURE  (ABNORMALITY  IN  POSTURE)  =  CAW  
CLAW  HAND   BUSMA  
BRACHIAL  PLEXUS  
ULNAR  NERVE  INJURIES  
SYRINGOMYELIN  
MUSCULAR  ATROPHIES  
ACUTE  POLIOMYELITIS  
APE  HAND   PAS  
PROGRESSIVE  MUSCULAR  ATROPHY  
AMYOTROPHIC  LATERAL  SCLEROSIS  
SYRINGOMYELIN  
WRIST  DROP   RPP  
RADIAL  NERVE  INJURIES  
POLIOMYELITIS  
POISONING  (LEAD,  ARSENIC,  ALCOHOL)  
2.  PALM  
ABNORMALITIES  
 
CAROTENODERMA   HEPATIC  DISEASE  
MYXEDEME  
THENAR  ATROPHY   MEDIAN  NERVE  
HYPOTHENAR  ATROPHY   ULNAR  NERVE  
HANSEN’S     THICK  AND  CORD  LIKE  
  41  
HYPOTHENAR  =  FLAT  AND  ATROPHIED  
+NUMBNESS  
PALMAR  ERYTHEMA   LLV  
LIVER  CIRRHOSIS  
LATE  STAGE  PREGNANCY  
VULVAR  HEART  DISEASE  
 
3.  FINGERS  
MALFORMATION  
POLYDACTYL  (SUPERNUMENARY  FINGER)   LAURENCE  –  BIEDL  SYNDROME  
ASSOCIATED  WITH:  
1. JUVENILE  OBESITY  
2. RETINAL  DEGENERATION  
3. GENITAL  HYPOPLASIA  
4. MENTAL  RETARDATION  
SYNDACTYL  (WEBBED  FINGERS)   CONGENITAL/HEREDITARY  
HEBERDEN’S  NODES  (OSTEOARTHRITIS)   OSTEOARTHRITIS  
PAINLESS  –  DIJ  
HARD  
OLD  WOMEN  
MEN  (DUE  TO  TRAUMA,  SINGLE  JOINT)  
HAYGARTHS  NODES  (RHEUMATOID  ARTHRITIS)   INFLAMMATORY  
MIDDLE  AND  PROXIMAL  
FUSIFORM  
PAINFUL  
JOINT  CAPSULE  THICKENED  
 
5.  DIGITS  
INFECTION  
PARONYCHIA   SWOLLEN  
REDDENED  
PAINFUL  
FELON   ABSCESS  TERMINAL  PULP  
ND
2  BACTERIAL  INFECTION  
ONSET  SWELL  AND  DULL  PAIN  
 
6.  FINGERNAILS  
MALFORMATION  
 
ONYCHORREXIS   MALNUTRITION  
-­‐ BRITTLE  NAIL  PLATE   IRON  DEFICIENCY  
-­‐ BORDER  FRAYED  AND  TORN   THYROTOXICOSIS  
  CALCIUM  DEFICIENCY  
CLUBBING  W/O  PERIOSTOSIS   TB  
-­‐ CONVEX   COPD  
ND
-­‐ 2  HYPOXIA   BROCHIECTASIS  
-­‐ PARROT-­‐HIPPOCRATIC-­‐SERPENT-­‐DRUMSTICK  
ABSENCE  OF  NAILS   ICHTHYOSIS  
-­‐ CONGENITAL   TRAUMA  
BITTEN  NAILS   PERSONALITY  DISORDER  
-­‐ IRREGULAR   NEUROLOGIC  
-­‐ SHORT  NAILS  
SQUARE  AND  ROUND   CAS  
ACROMGELY  
  42  
CRETINISM  
LONG  AND  NARROW   HEM  
HYPOPITIUTARISM  
EUNOCHOIDISM  
MARFAN’S  SYNDROME  
SPOON     HIS  
-­‐ CONCAVE   HYPOCHROMIC  ANEMIA  
-­‐ KOILONYCHOSIS   IRON  DEFICIENCY  
RARE:  RLS  
-­‐ RHEUMATIC  
-­‐ LICHEN  PLANUS  
-­‐ SYPHILIS  
EGGSHELL   VIT  A  DEFICIENCY  
-­‐ CONCAVE  
ND
ONYCHAUXIS   2  TO  FUNAL  INFECTION  
-­‐ HYPERTHOPHY   WASHER  WOMEN  
-­‐ NAILS  ON  TOP  OF  ANOTHER  
-­‐ IRREGULAR  DISCOLORED  
-­‐ UNKNOWN  CAUSE  
RED  HAFT  MOON   CARDIAC  
LANULA  RED  
AZURE  BLUE  HAFT  MOON   WILSON  
LANULA  BLUE  
ND
ONYCHOLYSIS   2  TO  FUNAL  INFECTION  AND  TRAUMA  
-­‐  SEPRATION  OF  NAIL  FROM  NAILBED   +FOUL  ODOR  
-­‐ THYROTOXICOSIS  
-­‐ ECZEMA  
-­‐ PSORIASIS  
-­‐ MYENTERIC  DISEASE  
SUB  UNGCAL  HEMORRHAGE   BACTERIAL  ENDOCARDITIS  
-­‐ SPLINTER  HEMMORAGE   TRICHINOSIS  
-­‐ LINEAR  RED  
-­‐ EMBOLIC  
BEUS’S  LINE   ACUTE  SEVERE  ILLNESS  
-­‐  TRANSVERSE  DEPRESSION  
 
7.  WRIST  –  ROM  DORSIFLEX,  PALMARFLEXION,  ULNAR  AND  RADIAL  DEVIATION  
CONDITIONS  
A.CHRONIC  ARTHRITIS    
–  PAINFUL  AND  FUSIFORM  ENLARGMENT  
B.  NON-­‐SUPPURATIVE  TENOSYNOVITIS    
–  PAINFUL  SWELLING  ANATOMIC  SNUFFBOX    
-­‐  (SAUSAGE  LIKE  SWELLING)  
-­‐  TENDON  SHEATHS  (EXTERNAL  POLLICIS  BREVIS  AND  ABDUCTOR  POLLICIS  LONGUS)  
-­‐  +CREPITUS  
-­‐  DUE  TRAUMA  AND  INFLAMMATION  (GOUT  AND  GONOCCOCAL)  
C.  CARPAL  TUNNER  SYNDROME  
  -­‐  COMPRESSION,  NEUROPATHY  MEDIAN  NERVE  
  -­‐  (+)  TPN  (TINGLING,  PAIN,  NUMBNESS)  
  -­‐  FLEXOR  CARPI  RADIALIS  AND  FLEXOR  CARPI  ULNARIS  
  -­‐  PHYSICAL  SIGN  
    ATROPHY  
    HYPERSTHESIAS  
    PROGESSIVE  WEAKNESS  AND  AWKWARDNESS  
  NOTE:  
  43  
1. TINEL’S  SIGN  –  TINGLING  PALMARIS  LONGUS  
2. PHALEN  SIGN  –  HYPEREXTENSION  BOTH  HANDS  FOR  3  MINS  (PRAYER  SIGN)  
 
8.  FOREARM  
-­‐  MOST  SUSCEPTIBLE  FRACTURE  
-­‐  VOLAR  MASS  FORM  BY  FLEXORS  
-­‐  SMITH’S  FRACTURE  
   
9.  ELBOW  
-­‐  SWELLING  MORE  COMMON  EXTENSOR  SURFACE  
-­‐  RHEUMATOID  NODULE  –  FOUND  OLECRANON  BURSAE  AND  ULNAR  REGION  
NOTE:    
HUMERO-­‐ULNAR  =  EXTENSION  AND  FLEXION  
HUMERO-­‐RADIAL  =  PRONATION  AND  SUPINATION  
   
  CUBITUS  VALGUS  =  OUTWARD  (ANGLE  LESS  THAN  170)  
  CUBITUS  VARUS  =  INWARD  
   
  OLECRANON  BURSITIS  =  STUDENT  MINER’S  
ND
  ARTHRITIS  ELBOW  =  TENNIS  2  TENDONITIS  
 
10.  SHOULDER  
 
WINGED  SCAPULA  
-­‐ SPRENGELS  SCAPULA  
-­‐ PARALYSIS  LONG  THORACIC  NERVE  
-­‐ CONGENITAL  
-­‐ SOMETIMES  ASSOCIATED  WITH  SHORT  WEBBLED  NECK  
 
11.  SPINE  
 
NODDING  AND  LIFTING  =  ATLANTO-­‐OCCIPITAL  
FLEXION  AND  EXTENSION  =  C3  AND  C7  
LATERAL  BENDING  =  MIDCERVICAL  
ROTATION  =  ATLANTO-­‐AXIAL  (C1-­‐C2)  
 
CONDITIONS:  
PANCOAST   -­‐  SUPERIOR  PULMO  SULCUS  SYNDROME  
-­‐  TUMOR  PULMO  APEX,  UPPER  MEDIASTINUM  AND  
SUPERIOR  THORACIC  APERTURE  
+NECK  PAIN  
PARESIS  OR  ATROPHY  OF  ARM  MUSCLE  MAY  OCCUR  
HORNER   UNI  MIOSIS,  PTOSIS  OF  EYELIDS  
-­‐SWEAT  
TUMOR  LUNG  APEX  OR  NECK  
CERVICAL  SPONDYLOSIS   DEGENERATION  OF  NUCLEUS  PULPOSUS  
CERVICAL  OSTEOARTHRITIS  
WHISPLASH   RUPTURE  OF  LIGAMENTUM  NUCHAE  
SUDDEN  FORCEFUL  HYPEREXTENSION  OF  NECK  WITH  
HYPERFLEX  RECOIL  
 
POST  TRAUMA  
HYPERREFLEXION  OF  NECK  –  C5  
PARTIAL  DISLOCATION  FROM  HYPEREXT  –  UNABLE  TO  NOD  
FRACTURE  ATLAS  –  SEVER  OCCIPITAL  HEADACHE  
  44  
 
THORACOLUMBAR  CONDITIONS  
1. WHIPLASH  CERVICAL  –  GRADUAL  NUMBNESS  AND  TINGLING  SENSATION  THAT  GOES  DOWN  TO  THE  HAND  
2. KYPHOSIS  –  FORWARD  DEFORMITY  OF  THORACIC  (HUNCHBACK)  
3. SMOOTH  CURVE  
4. ANGULAR  CURVE  -­‐  +GIBBUS  
5. LORDOSIS  –  POSTERIOR  CONCAVITY  OF  LUMBAR  SPINE,  DEEP  FURROW,  POT  BELLY  
6. SCOLIOSIS  –  FEMALE  WALK  SEXY,  CONGENITAL,  PARALYSIS  BACK  OR  ABDOMINAL  MUSCLE  
 
LOWER  EXT  
 
1.  HIP  JOINT  AND  THIGH  
 
TEST  FOR  HIP  
LESAQUE’S  TEST   STRAIGHT  LEG  RAISING  
SCIATIC  NERVE  
TEST  THE  RANGE  OF  HIP  FLEXION  
PATRICS   LATERAL  ROTATION  OF  THE  HIP  
GRAELEN’S   PASSIVE  HYPEREXTENSION  
ACTIVE  HYPEREXTENSION   ONLY  TEST  IN  PRONE  
 
2.  KNEE  
CONDITIONS  
GENU  VARUM   BOW  LEG  
LEGS  DEVIATE  TOWARD  MIDLINE  
1. RICKETS  
2. OSTEITIS  DEORMANS  
3. COWBOY  AND  JOCKEY  
GENU  VALGUM   KNOCKED  KNEE  
LATERAL  DEVIATION  OUTWARD  
GENU  RECURVATUM   KNEES  FIXED  IN  HYPEREXTENSION  WITH  LITTLE  ABILITY  
TO  FLEX  
ABSENSE  PATELLA  
 
1.  Which  of  the  following  disease  manifest  a  big  hand,  big  bones  and  muscles?  
a.  Gigantism  b.  Acromegaly  c.  Polydactyl  d.  Marfan  syndrome    
2.  What  is  the  action  of  axiohumeral  muscle  group?    
a.  Internal  rotation  of  shoulder    
b.  External  rotation  of  shoulder    
c.  Shrug  shoulder  at  the  back    
d.  All  of  the  above    
3.  What  is  the  action  of  axioscapular  muscles?  (same  choices)    
4.  This  disease  is  cause  of  adenoma  in  pituitary  gland  related  to  hypothyroidism?    
a.  Cretinism  b.  Ehler  danlos  syndrome  c.  mongolism    
5.  The  following  are  changes  seen  in  carpal  tunnel  syndrome?    
a.  Thenar  atrophy  b.  Hypothenar  atrophy  c.  Ape  hand  d.  All  of  the  above    
6.  Which  of  the  following  has  a  smooth  curve  of  spine?    
a.  Cancer  b.  Infectious  spondylitis  c.  Ankylosis  spondylitis  d.  Pott’s  disease    
  45  
7.  Which  of  the  following  has  an  angular  curve  of  spine?    
a.  Ostetitis  deformans  b.  Ankylosis  spondylitis  c.  Senile  osteoporosis  d.  Infectious  spondylitis  
8.  Manifestation  of  Achilles  tendon  damage  .  Positive  simmond  test  .  Body  weight  is  applied  to  base  of  the  foot  because  of  
plantar  flexion  .  Because  of  hyper  extension  and  dorsiflexion  .  A  and  b  only    
9.  This  is  associated  with  eunochoidism    
a.  Eggshell  nail  b.  Spoon  nails  c.  Long  narrow  nails  d.  Square  and  round  nails    
10.  Sub  ungal  haemorrhage  is  the  cause  of  the  following  disease  except?    
a.  Cardiac  disease  b.  SBE  c.  Trichinosis  d.  None  of  the  above    
11.  This  line  is  associated  with  acute  severe  illnesses  –  Beau’s  Line  This  is  associated  with  renal  disease  something?haha!  
forgot  exact  na  sakit  a.  Red  half  moon  b.  Blue  half  moon  c.  Onychauxis  d.  None  of  the  above    
12.  Test  for  lateral  rotation    
a.  Graenlen  b.  Stretching  of  legs  upward  c.  Patrick  d.  Owen  
Test  for  hip  flexion  (same  choices)  
13.  Type  3  pitting  edema  is?    
a.  2mm  b.  4mm  c.  6mm  d.  8mm    
14.  This  is  a  congenital  disease  associated  with  damage  to  long  thoracic  nerve?  –  Winged  scapula    
15.  Location  of  tumor  in  pancoast  syndrome    
a.  Apex  b.  Mediastinum  c.  Base  d.  A  and  b  e.  All  of  the  above    
16.  Responsible  for  movement  of  neck  from  left  to  right    
a.  C3-­‐c7  b.  Atlanto  occipital  c.  Axial  d.  Midcervical  vertebrae    
17.  Scoliosis  can  be?    
a.  Paralysis  of  back  muscle  b.  Congenital  c.  1  leg  is  paralyzed  d.  A  and  b  e.  All  of  the  above    
18.  Genu  recurvatum    
a.  Fixed  hyperextension  b.  Fixed  flexed  c.  Congenital  d.  A  and  c    
19.  Morrant  baker  cyst  is  associated  with?  –  Popliteal  artery  syndrome    
20.  Genu  Valgum?    
a.  Lateral  deviation  of  leg  b.  Bow  leg  c.  Cause  of  rickets  d.  .  ?    
21.  Genu  Varus    
a.  Can  be  cause  of  occupational  b.  Paget’s  disease  c.  Can  be  cause  by  rickets  d.  All  of  the  above    
22.  Associated  with  Marfan  Syndrome    
a.  Long  slender  nails  b.  Arachnodactyl  c.  Claw  hand  d.  A  and  b  e.  All  of  the  above    
23.  Cause  of  claw  hand  a.  Brachial  plexus  injury  b.  Syringomyelia  c.  Muscle  dystrophy  d.  All  of  the  above    
24.  Nodes  found  in  DIP  is  associated  with?    
a.  Osteoarthirits  b.  Systemic  rheumatic  arthritis  c.  Gout  arthritis  d.  Infectious  arthritis    
25.  Combing  of  hair  is  associated  with?    
a.  Internal  rotation  and  abduction  b.  External  rotation  and  abduction  c.  Internal  rotation  and  adduction  d.  External  
rotation  and  adduction    
26.  Movement  of  the  right  arm?    
  46  
a.  Counter  clockwise  on  pronation  and  clockwise  on  supination  b.  Counterclockwise  on  supination  and  clockwise  on  
pronation  c.  Both  are  correct  d.  None  of  the  above  
27.  What  should  you  assess  when  looking  at  the  spine  sideward?  
a.  Height  of  shoulder  
b.  Curvature  
c.  Location  of  ASIS  
d.  All  of  the  above  
 
28.  What  should  you  assess  if  you  are  looking  at  the  back  of  patient?  
a.  Height  of  shoulder  
b.  Dimples?  
c.  ASIS  
d.  All  of  the  above  
 
30.  If  a  person  cannot  raise  greater  than  10  degrees  in  elbow  joint?  
a.  Dislocation  
b.  Fracture  
c.  Damage  of  supraspinatus  tendon  
d.  All  of  the  above  
 
31.  Heloma  Durum  is?  
a.  Hard  corn  
b.  Callus  
c.  Soft  corn  
d.  Stiffened  toe  
 
32.  If  there  will  be  a  fracture  in  humerus,  what  should  you  check?  
a.  Brachial  artery  
b.  Radial  artery  
c.  Brachioradial  artery  
d.  ?  
Medicine  Extremities  
1. Dorsiflexion  –  Soleus  Tear  
2. Housemaid’s  –  anterior  knee  cyst  
3. To  test  range  of  hip  flexion,  nerve  irritation  and  lumbar  herniation  –  straight  leg  test  
4. Test  done  to  detect  excessive  lumbar  convexity  /  lordosis  –  Hugh  Owen  Thomas  sign  
5. Yellow  discoloration  in  palms  imparted  to  the  skin  by  carotene  –  carotenemia  
6. Inspection  of  vertebral  column  viewed  laterally  –  AOTA,  cervical,  thoracic  ,  lumbar  curve  
7. True  about  lordosis  except:  -­‐  posterior  convexity  of  lumbar  sign  
                 Posterior  concavity  of  lumbar  sign  
                 Deep  furrow  between  paraspinous  muscle  
                 Pot  belly    
8. Test  for  hip  dislocation  –  trendelenberg’s  sign  
9. What  is  the  joint  that  allows  free  movement  like  knee  and  shoulder  joint  –  fibrous  
         Synovial  
         Cartilaginous  
10. SITS  muscle  of  rotator  cuff  muscle  except:  -­‐  supraspinatus  
           Infraspinatus  
           Teres  minor  
           Subscapularis  Muscle  
           None  
11. ROM  of  ELBOW  –  extension  ,  pronation  flexion,  supination  
  47  
12. Drop  arm  definition  (bates)  -­‐  if  patient  cannot  hold  arm  fully  abducted  at  shoulder  level,  possible  rotator  cuff  
tear.  
13. This  can  be  seen  in  osteoarthritis    -­‐  heberdens  
 
14. Bouchards  -­‐  rheumatoid  arthritis  
           Asymmetric  deformitiesof  the  hands  and  wrists  
           All  
15. Range  of  motion  for  the  wrist:  flexion,  extension,  abduction,  adduction  
16. Hypertrophy  of  nailplates  caused  by  chronic  fungal  infection  –  onychauxis  
17. Long  narrow  nail  plates  except:  cretinism  
18. Pronated  wrist  drop  –  radial  nerve  palsy    
19. Rotation  of  neck  –  rotation  at  c1  and  c2  
20. Difference  in  shoulder  length  is  due  to  :  scoliosis  
Sprengels  shoulder  
          A  and  b  
          NONE  
21. View  person  from  spine  is  for  :  cervical  convexity  
                         Thoracic  convexity  
                               Lumbar  convexity  
                               None  
22. Matching  type:  hugh  owen  
23. Stretch  sciatic  muscle  –  straight  leg  raising  
24. Lateral  rotation  grip  –  patricks  test  
25. Passive  hyperextension  –  graelens  test  
26. Eversion  –  talipes  valgus  
27. Dorsiflexion,  defect  on  neural  arch  –  talipes  calcaneus  
28. Genu  varum  occur  in  –  rickettes  
                                                       Pagets  
                           Both  
29. Length  of  lower  extremities  –  from  ASIS  to  medial  malleolus  
30. Thumping  of  posterior  spine  –  volar  surface  of  wrist  
31. Spoon  nail  –  iron  deficiency  
32. Test  for  acromioclavicular  joint  –  cross  over  test  
33. Bowleg  –  genu  varum  
34. Bitten  nails  common  to  personality  disorder  
35. Knocked  knee  –  genu  valgum  
36. Onychauxis  –  chronic  fungal  infection  
37. Subungal  hemorrhage  –  splinter  hemorrhage  linear  red  hemorrhage  in  nailbed  
38. Square  and  round  nail  plates  –  cretinism  
 
Unequal  Leg  Length  can  be  seen  -­‐  Scoliosis,  Hip  Fracture,  Poliomyelitis    except:  Kyphosis    

 
 
 
 
 
 
 
  48  
 
EXTREMITIES  
1. Lumbar  concavity:    
 false  regarding  lordosis(dapat  lumbar  concavity)  
2. Soleus  tear:  
extreme  dorsiflexion  
3. Flexion  and  extension  occur  primarily  between  the  skull  and  C1,  the  atlas;  rotation  at  C1-­‐C2;  both  
4. Sits  muscles:    
supraspinatus,infraspinatus,teres  minor  and  subscapularis  
5. In  osteoarthritis,  Heberden’s  nodes  at  DIP  joints,  Bouchard’s  nodes  at  the  PIP  joint.    
 In  rheumatoid  arthritis,  symmetric  deformity  in  the  PIP,MCP  and  wrist  joints  with  ulnar  deviation.  
6. Wrist  movement:    
flex,extend,  abduct,  adduct  
7. Wrist  drop:    
radial  nerve  
8. Unequal  shoulder  heights  seen  in  scoliosis;:  
Sprengel’s  deformity  of  the  scapula(from  the  attachment  of  an  extra  bone  or  band  between  the  upper  scapula  
and  C7);  in  “winging”  of  the  scapula  (from  loss  of  innervations  of  the  serratus  anterior  muscle  by  the  long  
thoracic  nerve);  and  in  contralateral  weakness  of  the  trapezius  
9. Anterior  knee  cyst:    
prepatellar(housemaid’s  knee)  and  infrapatellar(clergyman’s  knee)  
10. Genu  varum:    
legs  deviate  towards  the  midline    
11. Long  narrow  nails:    
all  except  cretinism(included:  Marfan’s,  eunochoidism,  hypopituitarism)  
12. Palmar  erythema:    
except  hemolytic  anemia(included:  pregnancy,  portal  HPN,  vulvar  heart  dse)    
13. Motion  of  elbow:    
supination,  pronation,  flexion,  extension  
14. Thomas  sign:    
test  for  lumbar  lordosis  
15. Lesaque’s  test  
16. Synovial:  
 freely  movable(knee,  shoulder)  
17. Scoliosis,  kyphosis,  hip  fracture    
18. You  may  wish  to  percuss  the  spine  for  tenderness  by  thumping,  but  not  too  roughly  with  your  fist  
19. Patrick’s  test  
20. Dorsiflexion:    
either  ankle  motion  or  talipes  calcaneus  
21. Housemaid’s:  
anterior  knee  cyst  
22. To  test  range  of  hip  flexion,  nerve  irritation,  and  lumbar  herniation:    
straight  leg  test  
23. Test  done  to  detect  excessive  lumbar  convexity/lordosis:    
Hugh  owen  Thomas  sign  
24. Yellow  discoloration  in  palms  imparted  to  the  skin  by  carotene:    
carotenemia  
25. Inspection  of  vertebral  column  viewed  laterally:    
  49  
AOTA,  cervical,  thoracic,  lumbar  curve  
26. True  about  lordosis  except:    
posterior  convexity  of  lumbar  sign,  post  concavity  of  lumbar  sign,  deep  furrow  between  paraspinous  muscle,  
pot  belly  
27. Test  for  hip  dislocation:  
 trendelenberg’s  sign  
28. What  is  the  joint  that  allows  free  movt  like  knee  and  shoulder  joint:    
fibrous,  synovial,  cartilaginous  
29. SITS  muscle  of  the  rotator  cuff  muscle:    
supraspinatus,  infraspinatus,  teres  minor,  none  
30. ROM  of  elbow:      
extension,  pronation,  flexion,  supination  
31. Drop  arm  definition(BATES)  
32. This  can  be  seen  in  osteoarthritis:    
heberden’s,  bouchard’s,  asymmetric  deformities  of  the  hands  and  wrists,  all  
33. Range  of  motion  of  the  wrist:    
flexion,  extension,  abduction,  adduction  
34. Hypertrophy  of  the  nail  plates  caused  by  fungal  infection:  
onychauxis  
35. Pronated  wrist  drop:    
radial    nerve  palsy  
36. Rotation  of  the  neck:    
rotation  at  c1  and  c2  
37. Difference  in  shoulder  length  is  due  to:    
scoliosis,  sprengel’s  shoulder,  a  and  b,  none  
38. Long  narrow  nail  plates  except:  Cretinism  
39. View  person  for  spine  is  for:  cervical  convexity,  thoracic  convexity,  lumbar  convexity,  none  
40. Matching  type:  hugh  owen  
41. Stretch  sciatic  nerve:    
straight  leg  raising  
42. Lateral  roattion  grip:    
Patrick’s  test  
43. Passive  hyperextension:  
 graenlen’s  test  
44. Eversion:    
talipes  valgus  
45. Dorsiflexion,  defect  on  neural  arch:    
talipes  calcaneus  
46. Genu  varum  occurs  in:    
rickettes,  paget’s,    both  
47. Length  of  the  lower  extremities:    
from  ASIS  to  medial  malleolus  
48. Thumping  of  posterior  spine:    
volar  surface  of  wrist  
49. Spoon  nail:  
 iron  deficiency  
50. Test  for  acromioclavicular  joint:    
cross  over  test  
  50  
51. Bowleg:    
genu  varum  
52. Bitten  nails  common  to:    
personality  disorder  
53. Knocked  knee:  
 genu  valgum  
54. Onychauxis:    
chronic  fungal  infection  
55. Subungal  hemorrhage:    
splinter  hemorrhage  linear  red  hemorrhage  
56. Square  and  round  nailplates:    
cretinism  
57. Dorsiflexion:    
either  ankle  motion  or  talipes  calcaneus  
58. Test  done  to  detect  excessive  lumbar  convexity/lordosis:  
hugh  owen  Thomas  sign  
59. True  about  lordosis  except:    
posterior  convexity  of  lumbar  sign  
60. Test  for  hip  dislocation:    
trendelenberg’s  sign  
61. Joint  that  allows  free  movement  like  knee  and  shoulder  joint:    
synovial  joint  
62. ROM  of  elbow:  
 extension,  pronation,  flexion,  supnation  
63. Rotaion  of  neck:    
rotation  at  c1  and  c2  
64. Difference  in  shoulder  length:    
scloiosis,  sprengel’s  shoulder  
65. Lateral  rotation:    
patrick’s  test  
66. Passive  hyperextension:    
graenlen’s  test  
67. Eversion:    
talipes  valgus  
68. Stretch  sciatic  nerve:    
straight  leg  test  
 
 
 
 
 
 
 
 
 
 
 
 
  51  
1. One  of  the  following  regarding  examination  of  spine  is  incorrect  
a. From  the  side  inspect  the  spinal  profile  
b. From  behind  inspect  lateral  curves  
c. Percuss  the  spine  with  a  neurological  hammer  
d. Inspect  and  palpate  paravertebral  muscles  
2. Conditions  that  impair  range  of  motion  of  fingers  and  wrist  are:  
a. Arthritis  
b. Tenosynovitis  
c. Fibrous  in  the  palmar  fascia  
d. AOTA  
e. A  &  B  
3. All  of  the  following  are  associated  with  Marfan’s  syndrome  Except:  
a. Thumb  sign  
b. Hyperextensible  joints  
c. Hypertrophic  osteoarthropathy  
d. Elongated  long  bones  
4. The  following  statements  about  carpal  tunnel  syndrome  are  correct  Except:  
a. There  is  atrophy  of  the  radial  half  of  thenar  eminence  
b. Pain  especially  at  night  is  experience  
c. Weakness  and  loss  of  finer  movements  is  not  common  on  this  disease  
d. Trauma  from  excessive  flexion  of  the  wrist  is  common  cause  
5. Which  of  the  following  statements  is  associated  with  clawhand:  
a. Claw  is  formed  by  hyperextension  of  the  interphalangeal  joint  
b. The  condition  maybe  secondary  to  Rheumatoid  arthritis  
c. Brachial  plexus  injury  is  one  of  the  possible  cause  
d. AOTA  
6. Ruptured  Achilles  tendon  is  manifested  by:  
a. pain  at  the  heel  
b. inability  to  plantar  flex  the  foot  
c. inability  to  dorsiflex  the  foot  
d. shortening  of  the  tendon  
7. Pain  on  elevation  of  the  arm  60-­‐120  degrees  is  due  to:  
a. Chronic  tendinitis  
b. Partial  rupture  of  of  supraspinatous  tendon  
c. Dislocation  
d. Complete  fracture  
8. One  of  the  following  is  a  cause  of  kyphosis  in  post  menopausal  
a. Osteoporosis  
b. Faulty  posture  
c. Osteitis  deformans  
d. Ankylosing  spondylitis  
9. Gibbus  deformity  maybe  secondary  to:  
a. Pagets  dse  
b. Hypercalcemia  
c. Metastatic  carcinoma  
d. Cervical  spondylosis  
10. Internal  rotation  of  the  shoulder  joint  is  tested  by:  
a. Raising  both  arms  to  a  vertical  position  
  52  
b. Putting  both  hands  behind  the  small  ___  of  the  back  
c. Putting  both  hands  behind  the  neck  
d. NOTA  
st
11. Among  the  tests  of  the  hip  joint  the  most  gentle  test  that  should  be  done  1  is:  
a. Anvil  test  
b. Active  hyperextension  
c. Graenlen’s  test  
d. Rotation  of  the  thigh    
12. Deviation  of  the  legs  toward  the  midline  ,  leads  to  condition  called:  
a. Genu  valgum  
b. Bowlegs  
c. Knock  knee  
d. Genu  recurvatum  
13. Popliteal  artery  entrapment  syndrome  maybe  secondary  to:  
a. Prepatellar  bursitis  
b. Popliteal  abscess  
c. Clergyman’s  knee  
d. NOTA  
14. The  landmarks  of  the  shoulder  joint  are:  
a. Coracoid  process,  medial  epicondyle  and  greater  tuberosity  
b. Acromion,  coracoid  process,  greater  tuberosity  of  humerus  
c. Greater  tuberosity,  lateral  epicondyle,  clavicle  
d. NOTA  
15. Excessive  transverse  growth  of  the  nail  plate  causing  the  lateral  edge  to  lacerate  
a. Onychauxis  
b. Onychocryptosis  
c. Ram’s  horn  nail  
d. onychogryphosis  
 
 
 

 
 
 
 
 
 
 
 
 
 
 
 
 
 
 
 
  53  
 
1. Rheumatoid  Arthritis  
Answer Key
2. Acromegaly  
3. Mongolism   C
A
4. Palmar  Erythema   P
5. Phalen’s  Sign/  Tinel’s  Sign   J
M
6. Trendelenberg  test/  Anvil  test   H
B
7. Lesaque’s  sign  
I
8. Hugh  Owen  Thomas  Sign   D
L
9. Patrick’s  test   E
10. Winged  Scapula   N
Q
11. Talipes  Equinus   G
12. Talipes  Varus   K
13. Eggshell  nails  
14. Carotenoderma  
15. Thumb  sign/  Wrist  Sign  
 
A. Pituitary  tumor  
B. Straight  Leg  Raising  
C. PIPJ  
D. Lateral  Rotation  of  hip  
E. Toe  wear  of  shoes  
F. Short  fat  hand  
G. Yellow  vegetables  
H. Hip  dislocation  
I. Flexure  contracture  of  hip  joint  
J. Liver  Cirrhosis  
K. Arachnodactyly  
L. Sprengels  deformity  Median  Nerve  
M. Median  nerve  
N. Lateral  wear  of  shoes  
O. Active  hyperextension  
P. Little  finger  is  curved  
Q. Vitamin  A  deficiency  

 
 
 
 
 
 
 
 
 
 
 
 
 
 
 
  54  
 
 
 
1. Rheumatoid  Arrthritis/  Haygarth’s  Node   Answer Key

2. Acromicria   B
3. Mongolism   E
A
4. Palmar  Erythema     L
I
5. Tinel’s  Sign  
J
6. Simmond’s  test   G
F
7. Lesaques  sign   C
8. Hugh  Owen  Thomas  test   K
H
9. Graenlen’s  test   N
10. Azure  Half  moon   D
O
11. Whiplash  Injusry   P
12. Nodding  and  lifting  the  head  
13. Fracture  of  humeral  back  
14. Winged  scapula    
15. Olecranon  Bursitis  
 
A. Little  finger  is  curved  
B. PIPJ  
C. Passive  hyperextension  shoulder  pain  small  hand  
D. Flexion  contracture,  hip  joint  
E. Straight  leg  raising  
F. Ruptured  ligamentum  nuchae  
G. Median  nerve  
H. Achilles  tendon  
I. Hepatolenticular  regeneration  
J. Cirrhosis  
K. Short  thick  hands  
L. Atlanto  occipital  joint  
M. Paralysis  long  thoracic  nerve  
N. Miner’s  elbow  
O. Atlanto  axial  joint  

 
 
 
 
 
 
 
 
 
 
 
 
 
 
 
 
  55  
EXTREMITIES  
1.  Popliteal  artery  entrapment  syndrome  maybe  due  to:  
A.Housemaid’s  cyst      B.  Infrapatellar  cyst   C.  Popliteal  abcess   D.  Large  Baker’s  cyst  
2.  Splinter  hemorrhage  in  nailbeds  is  suggestive  of:  
A..SubAcute  bacterial  endocarditis          B.  Anemia       C.  Hepatolenticular  degeneration     D.  Iron  deficiency  
3.  Congenital  small  patella  
  A.Genu  Varum   B.  Genu  Recurvatum     C.  Genu  Valgus   D.  Degenerative  osteoarthritis  
4.  The  thumb  is  held  in  extension  by  its  inability  to  flex  
  A.Ape  hand   B.Claw  hand   C.Wrist  drop   D.  Benediction  Hand  
5.  In  performing  the  ROM  of  the  spine  the  examiner  should  assist  the  elderly  patient  by  
  A.Holding  one  hand  of  the  patient   B.Asking  him  to  be  careful  C.Asking  him  to  hold  Hand  Rails    
D.Stabilizing  the  pelvis  by  holding  the  pelvis  of  the  patient  while  patient  performs  ROM  
6.  Lesion  of  the  Ulnar  Nerve  leads  to:  
  A.Wrist  drop   B.  Claw  hand   C.  Hypothenar  atrophy   D.  Palmar  Erythema  
7.    As  the  examiner  positions  himself  behind  the  patient,  he  does  the  ff:  
  A.Inspect  for  lateral  curves   B.  Locate  the  dimples  of  venus   C.  Take  note  of  the  height  of  shoulders  and  iliac  crest  
  D.Inspect  cervical,  thoracic  and  lumbar  curves  
8.  When  looking  for  rheumatic  nodules  the  most  likely  area  to  look  is:  
  A.Medial  Epicondyle   B.  Extensor  surface  near  olecranon  process   C.Knee  Joint   D.  Lateral  Epicondyle  
9.  The  paravertebral  muscles  are  evaluated  by:  
  A.Inspection  for  fasiculations   B.  Ascultation  for  crepitus  C.  Palpate  for  tenderness  and  spasm   D.All  are  correct  
10.  Motions  of  the  forearm:  
  A.Pronation  &  Supination   B.  Flexion  &  Extension     C.  Both     D.  Neither  
11.  The  popular  theory  as  to  the  cause  of  clubbing  of  the  finger  is/are:  
  A.Floating  nail   B.-­‐20  degress  angulation  of  the  nail     C.hypozia   D.  All  
12.  Haygarth’s  Node  except  
  A.Distal  Interphalangeal  joint      B.  Rheumatoid  arthritis      C.Prodromal  Interphalengeal  joint    D.  Metacarphophalengeal  gout  
13.  A  casue  of  kyphosis  in  young  males  due  to  painful  disease  of  the  spine  is  
  A.Paget’s  Disease  B.  Osteoporosis   C.  Faulty  posture  D.Ankylosis  spondylitis  
14.  Legs  are  outwardly  deviated:  
  A.Genu  recurvatum   B.  Genu  varus   C.  Genu  valgum   D.  Osteoarthritis  Knee  
15.  Nodding  and  lifting  the  head  involves:  
  A.Midcervical  Vertebra   B.  C3-­‐C7  C.  Atlantooccipital  joint   D.  Atlantoaxial  joint  
16.  Conditions  that  impair    range  of  motion  of  the  hands  and  wrists  because  of  scarring  
  A.Arthritis   B.Dupuytren’s  Contracture   C.  Tendenitis   D.  ALL  
17.  Carpal  Tunnel  Syndrome:  
st nd rd th
  A.Numbers  if  the  Small  &  Ring  fingers  B.  Numbness  of  the  1 ,  2  ,3  &  4  fingers  C.  Wrist  drop  D.ALL  
18.  Ruptures  ligamentum  nuchae  
  A.C5  fracture   B.  Cervical  Spondylosis   C.  C1  fracture     D.  whiplash  cervical  injury  
19.  A  line  drawn  across  the  iliac  spine  approximates  the  
nd th th st
  A.2  lumbar  vertebra   B.4  lumbar  vertebra   C.7  thoracic  vertebra   D.1  lumbar  Vertebra  
20.  Absence  of  plantar  flexion  
  A.Tibial  Fracture     B.Ruptured  tendon  od  Achilles   C.  Ruptured  Baker’s  cyst   D.  Soleus  tear  
 
 
 
 
 
 
 
 
 
 
 
 
  56  
CHEST,  LUNGS,  BOOBS  
 
Medicine  I  
nd
Midterms  2  Semester  (February,  2013)  
 
1. Acute  onset  but  chronic  progressive.  
-­‐ SMOKING  
2. Which  is  likely  the  mechanism  of  cough  among  patients  with  heart  failure?  
-­‐ DECREASED  LUNG  COMPLIANCE  
3. Which  is  likely  the  mechanism  of  cough  among  patients  with  heart  failure?  
a. Pressure  in  the  airway  
b. Decreased  lung  compliance  
c. Inflammation  of  the  airway  
d. Thermal  change  
4. 50  y.o.  patient  male  with  >  5  months  cough.  He  cough  when  he  lays  down  wearing  tight  clothes.  It  is  productive  with  
watery  whitish  sputum.  
a. Congestive  heart  failure  
b. Gastroesophageal  regurgitation  
c. Bronchiectasis  
d. Intestinal  fibrosis  
5. 45  year  old  male  has  cough  for  3  months  now  and  claims  to  clear  his  throat  every  morning  
a. Smoking  
b. Post  Nasal  Drip  
8. A  62  years  old  female  was  found  unconscious  and  at  the  ER  she  was  noted  with  very  fast  and  deep  breathing  with  
fruity  breath.  Her  pattern  of  breathing  is:  
a. Biot’s  respiration  
b. Tachypnea  
c. Kassmaul’s  respiration  
d. Hyperpnea  
9. 17  yo  female  consulted  at  the  ER  because  of  high  grade  fever  w/o  other  symptoms.  She  is  observed  w/  abnormal  
respiration.  Her  abnormal  respiration  expected  to  be:  
a. Tachypnea  
b. Platypnea  
c. Hyperpnea  
d. Kussmaul’s  
10.  An  IV  drug  user  presents  to  ER  due  to  hemoptysis  and  DOB  but  denies  any  fever  and  cough.  What  is  the  most  
common  cause  of  hemoptysis?  
a. PTB  
b. Lung  Abscess  
c. Pneumonia  
d. Vasculitis  
12. Kussmaul’s  respiration  is  heard  among  patient  with:  
a. Meningitis  
b. Ascites  
c. CVD  
d. Renal  failure  
13. 38  y/o  female,  rushed  to  ER  because  of  severe  retrosternal  pain,  relieved  by  nitroglycerine;  pain  occurs  when  cold  
substance  ingested.  
a. Angina  
b. Trachitis  
c. Esophagitis  
d. Esophageal  spasm  
14. Acute  onset  of  Difficulty  of  Breathing  precipitated  by  valsalva  maneuver  
a. Pulmonary  edema  
b. Rupture  of  aortic  aneurism  
  57  
c. Pneumothorax  
d. Pulmonary  emboli  
15. Most  common  cause  of  hemoptysis  
-­‐ Bronchiectasis  
16. Patient  with  heart  failure  
-­‐ Orthopnea  
17. Dry  cough  and  unilateral  effusion  
-­‐ Trepopnea  
18. Pneumococoniosis  is  due  to  exposure  to:  
a. Silica  
b. Coal  
c. Asbestos  
d. Silver  
19. Bagasse  
-­‐ Sugar  cane  
20. Primary  malignancy  
-­‐ Silica  
21. Psittacosis  
-­‐ Parrot  
22. Lagging  of  one  side  of  the  chest  is  best  identified  by:  
-­‐ Palpation  
24. Where  is  the  landmark  for  the  posterior  rib?  
a. C7  
b. T1  
c. Superior  scapular  line  
d. Inferior  scapular  line  
26. Referenced  used  in  identifying  posterior  ribs  
a. C7  
b. T1  
c. Superior  scapular  line  
d. Inferior  scapular  line  
27. Tactile  fremitus  is  usually  decreased  with:  
a. Atelactasis  
b. Emphysema  
c. Bronchitis  
d. Pneumonia  
29. Stridor  
-­‐ Epiglottis  
31. Normal  breath  sound  heard  in  intrascapular  area  
a. Vesicular  
b. Bronchovesicular  
c. Bronchial  
d. Tracheal  
33. Sounds  of  secreations  produce  on  bronchi?  
a. Stridor  
b. Rhonchi  
c. Rales  
d. Wheezes  
34. Bilateral  hyperresonance  
a. Tension  pneumothorax  
b. Pneumomediastinum  
c. Chronic  bronchitis  
d. Emphysema  
 
 
  58  
35. Findings  on  the  exam  of  the  anterior  chest  would  mostly  involved  which  of  the  following:  
a. Upper  lobe  
b. Middle  lobe  
c. Lower  lobe  
d. A  &  B  
36. Shorter  I:E  ratio  
a. Metabolic  acidosis  
b. Bronchitis  
c. Pleurisy  
d. Fever  
37. Describe  the  type  of  breathing  for  patient  with  heart  failure:  
a. Cheyne-­‐stokes  
b. Biot  
c. Kusmail  
d. Apneustic  
38. Fremitus  is  decreased  
a. Pneumonia  
b. Obesity  
c. Pleural  effusion  
d. Atelectasis  
40. Early  inspiratory  crackles  is  heard  at:  
a. Interstitial  lung  disease  
b. Pneumonia  
c. CHF  
d. Chronic  bronchitis  
41. Breast  extends  from?  
nd th
-­‐ 2  to  6  ICS  
42. Most  frequently  palpable  lymph  node  in  Breast  Ca:  
-­‐ Central  Lymph  Node  
43. Mastitis  
-­‐ Lactation  
44. Dividing  into  quadrants  is  toL  
a. Clinical  finding  
b. Staging  
46. Seen  in  acute  onset  and  chronic  progressive  DOB:  
a. Bronchiectasis  
b. Asthma  
c. Pneumonia  
d. Anemia  
47. Orange  peel  in  breast  cancer:  
a. Protrusion  of  mass  
b. Lymphatic  abscess  
c. Venous  abscess  
d. Increased  blood  flow  
 
 
Q:  Mediastinal  mass  cough,  described  as  signs  of  respiratory  distress,  except:  
 
Q:  Diaphragmatic  excursion  except:  
a. Atelectasis  
b. Pneumonia  
c. Pneumothorax  
d. Pleural  effusion  
 
Q:  A  35  y/o  consulted  for  fear  of  breast  cancer.  Her  sister  died  of  breast  cancer  at  23  y/o.  What  is  the  risk?  
  59  
a. >40  
b. 2.1-­‐4  
c. 1.1-­‐2  
d. None  
 
Q:  Suggestive  of  barky  cough  
 
Q:  Brassy  cough  
A:  Emphysema  
 
Q:  Brassy  
A:  Trachea  
 
Q:  Barky  
A:  Glottis  
 
Q:  Pink  puffer  
A:  Emphysema  
 
Q:  Mastitis  
A:  Lactation  
 
Q:  Man  can’t  keep  up  with  friend  at  same  age  
A:  Grade  2  
 
Q:  Occupational  asthma  common  among  workers  exposed  to  which  of  the  following  dust:  
a. Asbestos  
b. Coal  
c. Cotton  dust  
d. Silicon  
 
Q:  Pneumocosis  
A:  Coal  
 
Q:  Heart  Failure  
A:  Orthopnea  
 
Q:  Apex  above  clavicle  
A:  2-­‐4cm  
 
Q:  COPD  
A:  10x  
 
Q:  Lung  cancer  in  women  
A:  13x  
 
 
Q:  Basilan  
A:  Paragoniasis  
 
Q:  Rusty  color  of  sputum  
A:  S.  pneumonia  
 
Q:  Pursed  lip  
A:  Emphysema  
  60  
 
Q:  Crescendo=decrescendo  with  apnea  
A:  Cheyne  strokes  
 
Q:  Afferent  limb  
A:  IX  
 
Q:  Decreased  or  low  fremitus  
A:  Thick  chest  wall  
 
Q:  Soft  and  low  
A:  Vesicular  
 
Q:  Early  inspiratory  crackles  
A:  Chronic  bronchitis  
 
Q:  Most  common  palpable  lymph  node  of  breast  cancer  
A:  Central  Node  
 
Q:  High  (4x)  breast  density  
A:  Highest  to  relative  risk  for  breast  cancer  
 
Q:  Modifiable  risk  for  breast  cancer  
A:  Obesity  
 
Q:  Chronic  cough  
A:  2  months  
 
Q:  Cough  complication  
A:  Pneumothorax  
 
Q:  IE  ration  is  decreased  in  fever  
A:  Chronic  bronchitis  
 
Q:  Chest  pain  associated  with  chest  tenderness  pathology  of  
A:  Chest  wall  
 
Q:  Pursed  lip  breathing  is  manifested  by  patients  with:  
a. Bronchiectasis  
b. Asthma  
c. Emphysema  
d. Pneumonia  
 
Q:  Pink  puffer  also  known  as  
a. Asthma  
b. Emphysema  
c. Chronic  bronchitis  
 
Q:  Below  are  hereditary  diseases  except  
a. HPN  
b. Leprosy  
c. Heart  Disease  
d. DM  
 
 
  61  
 
BREAST    
1. Intraductal  Papilloma  
 >  SPONTANEOUS  UNILATERAL  BLODDY  DISHARGE  FROM      ONE/TWO  WARANTS  FURTHER  EVALUATION  
2. 1.1–  2.0    
>  OBESITY,  LATE  AGE  ETC.  
3. Surface  of  Areola    
>  SMALL,  ROUNDED  ELEVATION  FORMED  BY  SEBACCEOUS  GLANDS      ,  SWEAT  GLANDS  &  ACCESSORY  AREOLAR  
GLANDS  
4. Tail  of  Spence    
>  EXTENDS  ACROSS  THE  ANTERIOR  AXILLARY  FOLD  
5. Female  Breast    
ND TH
>  CLAVICLE,  2  RIB  DOWN  TO  THE  6  RIB  AND  FROM  THE  STERNUM  ACROSS  TO  THE  MIDAXILLARY  LINE  
 
 
Recall  –  Clinical  Medicine  “Chest  and  Lungs”  (Dec.  17,  2012)  
 
1.  During  cough,  the  speed  of  airflow  in  the  airway  is  as  fast  as  the  speed  of:  
  a.  Light           c.  Airplane  
  b.  Sound         d.  Lightning  
 
2.  What  is  etiology  of  cough  in  pleural  effusion?  
  a.  Inflammation         c.  Decrease  pulmonary  compliance  
  b.  Mechanical  irritation       d.  Chemical/thermal  
 
3.  18  y.o.  female  with  acute  onset  of  cough  associated  with  runny  nose,  sore  eyes,  and  chest  discomfort.  What  condition?  
  a.  Pneumonia         c.  Mycoplasm  pneumonia  
  b.  Acute  bronchitis       d.  Tracheobronchitis  
 
4.  Chronic  cough,  sputum  purulent,  often  copious  and  foul-­‐smelling  
  a.  Bronchiectasis       c.  Obstructive  pneumonia  
  b.  Chronic  bronchitis       d.  PTB  
 
5.  Cause  of  brassy  cough  
  a.  Glottis         c.  Bronchiole  
  b.  Trachea         d.  Lung  parenchyma  
 
6.  Patient  with  which  of  the  following  lung  problem  would  present  with  sudden  onset  of  difficulty  of  breathing  preceded  
by  pleuritic  chest  pain?  
  a.  Pulmonary  edema       c.  Pulmonary  emboli  
  b.  Pneumothorax       d.  Pulmonary  hemorrhage  
 
7.  Sudden  onset  of  difficulty  of  breathing  precipitated  by  exposure  to  grass  pollen  
  a.  Asthma         c.  Pneumonia  
  b.  COPD         d.  Hypersensitive  Pneumonitis  
 
8.  Alpha  I  antitrypsin  increase  risk:  
  a.  Asthma         c.  Chronic  bronchitis  
  b.  Cystic  fibrosis         d.  Emphysema  
 
9.  49  y.o.  DB,  he  has  DOB  for  3  years.  Lately,  he  can’t  keep  up  with  his  friends  of  the  same  age.  What  is  the  grade  or  
American  Thoracic  Dyspnea  Scale?  
  a.  Grade  1         c.  Grade  3  
  b.  Grade  2         d.  Grade  4  
 
  62  
10.  Considered  as  the  most  common  cause  of  hemoptysis?  
  a.  Pneumonia         c.  Bronchiectasis  (not  sure)  
  b.  PTB           d.  Lung  CA  
 
11.  Lung  parenchyma,  common  cause  of  hemoptysis?  
  a.  Bronchiectasis         c.  Bronchogenic  CA  (not  sure)  
  b.  Good  pasteur’s  syndrome     d.  Cystic  fibrosis  
 
12.  Squeezing  retrosternal  pain  felt  after  ingestion  of  cold  liquid  and  relieved  by  nitroglycerin  is  felt  in?  
  a.  Heart           c.  Chest  Wall  
  b.  GIT           d.    
 
13.  40  y.o.  female  with  rheumatoid  arthritis  of  more  than  15  years  now  and  taking  methotrexate.  Started  to  have  cough  
and  difficulty  of  breathing  2  years  ago.  What  respiratory  disease  is  the  most  likely  the  cause?  
  a.  Non-­‐cardiogenic  pulmonary  edema   c.  Pulmonary  vasculitis  
  b.  Pulmonary  emboli       d.  Interstitial  infiltrative  disease  
 
14.  Which  of  the  following  occupational  inorganic  substances  will  cause  pneumoconiosis?  
  a.  Bagasse         c.  Coal  
  b.  Asbestos         d.  Silica  
 
15.  Exposure  to  which  of  the  following  dusts  increases  the  risk  of  lung  cancer?  
  a.  Bagasse         c.  Coal  
  b.  Tin  oxide         d.  Silica  
 
16.  Cigarette  smoking  increases  risk  of  COPD  mortality  by:  
  a.  2.3x           c.  10x  
  b.  4x           d.  13x  
 
17.  Smoking  increases  lung  cancer  mortality  in  men  by:  
  a.  4x           c.  20x  
  b.  6x           d.  23x  
 
18.  Alcoholics  have  higher  risk  of  getting  pneumonia  and  they  are  more  prone  to  develop:  
  a.  Aspiration  pneumonia       c.  Pneumonia  due  to  tularemia  
  b.  Psittacosis         d.  Pneumocystic  pneumonia  
 
19.To  decrease  tension  pneumothorax  where  will  you  insert  the  big  bore  needle:  
nd th
  a.  2  ICS  MCL  (Mid  Clavicular  Line)   c.  6  ICS  AAL  (Anterior  Axillary  Line)  
th th
  b.  4  ICS  MAL  (Mid  Axillary  Line)   d.  8  ICS  MAL  
 
20.  Which  of  the  following  vertebrae  has  the  most  prominent  spinous  process  
  a.  C6           c.  T1  
  b.  C7           d.  
 
21.  Needle  insertion  of  thoracocentesis  
nd th
  a.  2  ICS         c.  6  ICS  
th th
  b.  4  ICS         d.  8  ICS  
 
22.  The  apex  of  the  lungs  is  extended  above  the  inner  1/3  of  the  clavicle  by  as  much  as:  
  a.  1-­‐3cm           c.  3-­‐5cm  
  b.  2-­‐4cm         d.  4-­‐6cm  
 
23.  Diaphragmatic  excursion  from  resting  to  full  inspiration  is  at:  
  a.  1-­‐3cm           c.  5-­‐6cm  
  63  
  b.  3-­‐5cm           d.  6-­‐7cm  
 
24.  Bifurcation  of  trachea  at:  
nd
  a.  Manubrium         c.  2  ICS  
rd
  b.  Sternal  Angle         d.  3  Rib  
 
25.  Pursed  lip  breathing  
  a.  Emphysema         c.  Asthma  
  b.           d.  
 
26.  Blue  bloater  
  a.  Chronic  bronchitis       c.  Bronchiectasis  
  b.  Emphysema         d.    
 
27.  Ribs  angle  is  usually  more  horizontal  compared  to  normal  individual  if  patient  have  which  of  the  following  problem:  
  a.  Pneumonia         c.  Pneumothorax  
  b.  COPD         d.  Mild  Asthma  
 
28.  Normal  angle  of  the  rib:  
  a.  35  degrees         c.  45  degrees  
  b.  40  degrees         d.  50  degrees  
 
29.  Narrowing  of  ICS  may  cause  shift  to  the  chest  to  the  one  side:  
  a.  Massive  effusion       c.  Disc  atelectasis  
  b.  Severe  pneumonia       d.  Obstruction  of  main  bronchus  
 
 
30.  Unilateral  widening  and  even  bulging  of  ICS  is  suggestive  of:  
  a.  Massive  effusion       c.  Large  lung  mass  
  b.  Severe  pneumonia       d.  Pneumothorax  
 
31.  IE  ratio  in  emphysematous  patient:  
  a.  1:2           c.  1:4  
  b.  1:3  (not  sure)         d.  1:5  
 
32.  Which  among  the  following  abnormal  respiratory  rhythm  is  seen  in  the  lesion  of  the  PONS?  
  a.  Cheyne  stroke         c.  Kussmaul  
  b.  Biot’s           d.  Apneustic  
 
33.  Diabetic  Ketoacidosis:  
  a.  Cheyne  stroke         c.  Kussmaul  
  b.  Biot’s           d.  Apneustic  
 
34.  Type  of  respiration  associated  with  morphine  overdose:  
  a.  Cheyne  stroke         c.  Kussmaul  
  b.  Biot’s           d.  Apneustic  
 
35.  Clubbing  of  fingers  are  seen  in  this  pathologic,  except:  
  a.  Bronchiectasis         c.  Mesothelioma  (not  sure)  
  b.  COPD         d.  Pulmonary  fibrosis  
 
36.    
37.  Which  of  the  following  breath  sounds  have  intermediate  pitch  and  loudness?  
  a.  Tracheal         c.  Broncho-­‐vesicular  
  b.  Bronchial         d.  Vesicular  
  64  
 
38.  Which  of  the  following  breath  sounds  has  loud  and  high  pitched  sound?  
  a.  Tracheal         c.  Broncho-­‐vesicular  
  b.  Bronchial         d.  Vesicular  
 
39.    
40.  Mid  inspiratory  &  expiratory  crackles  usually  heard  in  patients  with:  
  a.  Bronchiectasis  
 
41.  Normally  breast  tissue  extend  from  the  sternum  to:  
  a.  AAL           c.  PAL  
  b.  MAL           d.  Between  AAL  &  MAL  
 
42.  Rounded  elevations  seen    in  the  areola  is  caused  by  the  following  except:  
  a.  Sebacceous  glands       c.  Accessory  glands  
  b.  Sweat  glands         d.  Fat  globule  
 
43.  Fibroadenoma  of  the  breast  is  a  common  cause  of  breast  mass  among  female  aging:  
  a.  15-­‐20  y.o.         c.  50-­‐65  y.o.  
  b.  25-­‐50  y.o.         d.  >65  y.o.  
 
44.  40  y.o.  nullipara,  married,  no  family  history  of  breast  CA,  what  is  the  risk  factor?  
  a.  >4           c.  1.1-­‐2.0  (not  sure)  
  b.  1-­‐4           d.  
 
45.  Which  of  the  following  can  decrease  the  risk  of  breast  CA?  
  a.  Use  of  contraceptive       c.  Breast  feeding  
  b.  Late  pregnancy         d.  Post  menopausal  
 
46.  MRI  of  the  breast  is  recommended  for  the  following  except:  
  a.  Older  woman         c.  High  risk  group  
  b.  Newly  diagnosed  breast  CA     d.  Women  with  dense  breast  
 
47.  Galactorrhea  may  be  seen  in  patient  with  
  a.  Herpes  zoster         c.  Ovarian  CA  
  b.  Pituitary  adenoma       d.  
 
48.  Benign  condition  of  breast  may  cause  bloody  breast  discharge  
  a.  Adenofibrosarcoma       c.  Fibroadenoma  
  b.  Intraductal  papilloma       d.  Neurosarcoma  
 
49.  Breast  malignancy  is  most  likely  to  have  if  breast  tumor  is  about:  
  a.  2cm           c.  4cm  
  b.  3cm           d.  5cm  (not  sure)  
 
50.  Breast  malignancy  is  highly  considered  if  the  patient  have    
  a.  Smooth  border         c.  Palpable  mass  
  b.  Wall  delineated       d.  Immobile  
 
 
 
 
 
 
 
  65  
CARDIOLOGY  
 
Areas  of  Auscultation  
nd
a. 2  ICS  LSB  
nd
b. 2  ICS  RSB  
c. Erb’s  Point  
th
d. 4  ICS  LSB  
th
e. 5  ICS  LMCL  
nd
36.  Pulmonary  Stenosis  –  2  ICS  LSB  
th
37.  Mitral  Stenosis  –  5  ICS  LMCL  
nd
38.  Opening  Snap  –  2  ICS  LSB  
th
39.  Mitral  valve  prolapse  –  5  ICS  LMCL  
nd
40.  PDA  –  2  ICS  LSB  
nd
41.  Aortic  stenosis  –  2  ICS  RSB  
th
42.  S3  &  S4  –  5  ICS  LMCL  
th
43.  Mitral  regurgitation  –  5  ICS  LMCL  
nd
44.  Splitting  of  S2  –  2  ICS  LSB  
th
45.  Tricuspid  regurgitation  –  4  ICS  LSB  
 
 
 
 
 
 
 
Clinical  Medicine:  CVS  exam  
1  to  5  choices:  
a.  concentric     d.  RVH  
b.  eccentric     e.  LVH  
c.  LV  dilatation  
 
1.  Not  palpable  in  PE  
2.  Forceful  apical  beat  but  not  displaced  
3.  Markedly  displaced  at  the  left  axillary  line,  hardly  palpable  
th th
4.  Displaced  to  axillary  line  5  or  6  ICS  anterior  axillary  line  
5.    Palpable  parasternally  but  retract  in  systole  
 
6  to  20  choices:  
nd th
a.  2  Left  ICS     d.  4  ICS  PS  
nd th
b.  2  Right  ICS     e.  5  ICS  MCL  
c.  Erb’s  point  
 
D   6.  Pulmonic  stenosis  
C   7.  VSD  
D   8.  ASD  
A   9.  PDA  
10.    
11.  Aortic  valve  regurgitation  
E   12.  Mitral  valve  regurgitation  
D   13.  Mitral  prolapse  
B   14.  Aortic  stenosis  
E   15.  Mitral  stenosis  
E   16.  Loud  S1  
A   17.  Opening  snap  
  66  
A   18.  Loud  P2  
D   19.  Hypertrophic  cardiomyopathy  
E   20.  LV  hypertrophy  
 
Multiple  choice  ito  na  super  haba,  eto  yung  summary  
21.  Mitral  stenosis  –  diastolic  rumbling  
22.  Aortic  stenosis  –  transmitted  to  the  carotid  
23.  Aortic  regurgitation  –  Diastolic  rumbling  murmur  
24.  VSD  –  pansystolic  
 
25  to  30  choices:  
a.  Caravallo’s  sign     d.  Gallavardin’s  
b.  Kussmauls       e.  Austin-­‐Flint  
c.  (+)  Hepatojugular     f.  Machinery-­‐like  
 
D   25.  Aortic  stenosis  
A   26.  Tricuspid  regurgitation  
F   27.  PDA  
E   28.  Chronic  aortic  regurgitation  
C   29.  CHF  
B   30.  Constrictive  pericarditis  
 
31  to  35  choices:  
a.  Loud  P2     d.  Opening  snap  
b.  Loud  A2     e.  Ejection  Click  
c.  Non-­‐ejection  sound  
 
D   31.  Mitral  stenosis  
C   32.  Mitral  valve  prolapse  
33.  Constrictive  pericarditis  
A   34.  Pulmonary  hypertension  
B   35.  Chronic  uncontrolled  hypertension  
 
36  to  40  choices:  
a.  Paradoxical  
b.  Wide  
c.  Loud  
d.  Negative  hepatojugular  reflex  
 
36.  Mitral  stenosis  
37.  Hypertensive  urgency  
38.  Pulmonary  hypertension  
A   39.  Severe  aortic  stenosis  
40.  Pulmonary  hypertension  
 
 
 
 
 
 
 
 
 
 
 
  67  
41  to  45  choices:  
a.  Pulsus  parvus  et  tardus  
b.  Corrigan  pulse  
c.  Pulsus  bisferiens  
d.  Hepatojugular  
e.  Neck  vein  distention  at  60˚  
 
D   41.  Normal  PE  finding  
E   42.  Pulmonary  edema  
A   43.  Aortic  stenosis  
C   44.  Combined  aortic  regurgitation  and  aortic  stenosis  
45.  Chronic  aortic  regurgitation  
 
46  to  50  choices:  
a.  Ejection  systolic  murmur  best  heard  at  
b.  Persistent  splitting  
c.  Diastolic  murmur  
d.  Rumbling  
e.  Blowing  
 
46.    
A   47.  Aortic  stenosis  
D   48.  Mitral  stenosis  
49.Aortic  regurgitation  
50.  Pulmonic  stenosis  
 
 
CVS  
 
A.Concentric  
B.  RVH  
C.  Kussmaul  Sign  
D.  +Hepatojugular  Reflex  
E.  LV  Dilatation  
 
 
 
 
 
 
 
 
 
 
 
 
 
 
 
 
 
 
 
 
 
  68  
1.  52  year  old  female  known  to  be  diabetic  and  hypertensive  patient  admitted  because  of  severe  dyspnea  (Orthopnea),  
cardiomegaly  and  bilateral  basal  crackles.  Neck  vein  is  visible  and  become  more  distended  upon  application  of  pressure  in  
the  liver  
 
2.   27   year   old   male   complained   of   fever,   dyspnea   and   chest   pain   aggravated   by   breathing.   Apical   beat   is   markedly  
displaced  to  midaxillary  region  with  distant  heart  sound.  The  patient  is  diagnosed  with  viral  cardiomyopathy.  
 
3.   42   year   old   male   who   was   not   aware   that   he   was   hypertensive,   when   seen   in   OPD   his   BP   was   180/110   on   sitting  
position.  Apex  beat  at  5th  ICS  LAAL  with  apical  beat  to  be  sustained  and  foreceful.  
 
4.   36   year   old   female   school   teacher   with   history   of   frequent   sore   throat   was   admitted   because   of   pedal   edema,   esay  
fatigability  and  DOE.  +Parasternal  retraction  during  systole  and  an  apical  diastolic  rumbling  murmur  
 
5.   65   years   old   male   known   CAD   patient   status   post   bypass   graft   surgery   10   years   ago   is   now   suffering   from   fatigue,  
dyspnea,  orthopnea,  edema  and  pulmonary  edema.  Apical  beat  is  hardly  palpable  at  6th  ICS.  Axillary  region  faintly  audible  
heart  sounds  and  adynamic  precordium  
*****  
 
A.  Loud  S1  
B.  Soft  S1  
C.  Loud  A2  
D.  Loud  P2  
 
1.  Chronic  regurgitation  
2.  Hypertensive  Urgency  with  BP  220/120  
3.  Rheumatic  Mitral  Stenosis  
4.  Pulmonary  edema  in  a  patient  with  CHF  
5.  Chronic  mitral  regurgitation  
*****  
 
A.  Physiologic  Splitting  S2  
B.  Persistent  Splitting  S2  
C.  Paradoxical  Spilitting  S2  
D.  Wide  Splitting  S2  
 
1.  ASD  
2.  Pulmonic  Stenosis  
3.  Aortic  Stenosis  
4.  Complete  RBBB  
5.  LBBB  
****  
 
 
 
 
 
 
 
 
 
 
 
 
 
 
  69  
A.  Right  mitral  stenosis  
B.  Right  aortic  with  mitral  regurgitation  
C.  Congenital  bicuspid  aortic  stenosis  
D.  Congenital  pulmonic  stenosis  
E.  Right  aortic  regurgitation    
 
1.  36  year  old  male  admitted  because  or  bilateral  pedal  edema  and  orthopnea.  P.E  revealed  a  hyperdynamic  apical  pulse  
seen  adn  palpable  at  6th  ICS  LMA  region  and  a  diastolic  blowing  murmur  at  the  ERBS  point  transmitted  to  the  apex  
 
2.  A  42  year  old  GRO  was  admitted  because  of  easy  fatigability  pedal  edema  orthopnea  and  PND  which  started  around  6  
months  PTC  becoming  progressive.  P.E  revealed  diastolic  thrill  and  murmur  with  Loud  S1  and  opening  snap  
 
3.  42  year  old  male  admitted  because  of  chest  pain  and  episode  of  syncope.  P.E  revealed  +ejection  systolic  murmur  at  2nd  
ICS  RSB  transmitted  to  the  neck  
 
4.  30  year  old  female  presents  a  persistent  spilitting  of  S2  with  expiratory  splitting  of  S2  at  the  2nd  ICS  LCB  ejection  systolic  
murmur  at  the  same  time  
 
5.   42   year   old   male   known   RHD   patient   was   admitted   because   of   severe   dyspnea,   jaundice,   ascites,   hepatomegaly,   P.E  
revealed   apical   beat   markedly   displaced   at   the   6th   ICS   left   midaxillary   region   with   apical   lift,   apical   diastolic   blowing  
murmur  transmitted  from  the  axilla  and  posterior  scapula  
 
Significant  Clinical  Findings  
1. Important  physical  examination  finding  in  CHF  –  (+)  hepatojugular  reflux  
2. Systolic  ejection  murmur  of  chronic  severe  aortic  regurgitation  is  accompanied  by  which  sound  –  Austin-­‐Flint  
murmur  
3. Mid-­‐diastolic  murmur  heard  over  the  pulmmonic  area  is  ASD  is  accompanied  by  which  sound  –  Persistent  
splitting  of  S2  
4. Increased  loudness  of  holosystolic  murmur  of  tricuspid  regurgitation  upon  inspiration  –  Caravallo’s  sign  
5. Heard  in  aortic  stenosis  –  Paradoxical  splitting  of  S2  
 
Cardiomyopathies  
th
6. Apex  beat  is  displaced  at  the  6  ICS  LAAL,  (+)  para  sterna  heave,  (+)  lift  –  Eccentric  
7. 40  year  old  female,  with  apical  beat  that  retracts  during  systole  –  Right  Ventricle  Hypertrophy  
8. Hardly  palpable  –  Left  ventricle  dilatation  
th
9. 40  year  old  female  with  chronic  hypertension  complained  of  chest  pain,  PE  revealed  apex  beat  at  the  5  ICS  LMCL  
with  a  (+)  apical  lift  –  Concentric  
10. A  52  year  old,  Male,  chronic  hypertensive  and  DM2  complained  of  chest  pain  and  SOB  (DOB?),  PE  revealed  (+)  
th
murmur  of  AR  and  MR,  apex  beat  is  displaced  at  5  ICS  left  anteroaxillary  line  with  (+)  lift.  –  Eccentric    
 
Abnormal  Sounds  
11. Pulmonary  Hypertension  –  loud  P2  
12. Mitral  stenosis  –  Loud  S1  
13. Atrial  Fibrillation  –  Soft  S1  
14. BP  220/120  –  loud  S1  
15. Pulmonic  stenosis  –Paradoxical  splitting  of  S2  
16. Sinus  Tachycardia  –  loud  S1  
17. Chronic  aortic  regurgitation  –  Loud  A2  
18. LBBB  –  Paradoxical  splitting  of  S2  
19. Severe  aortic  stenosis  –  Loud  A1  
20. Normal  PE  findings  –  splitting  of  S2  upon  inspiration  
 
 
 
 
  70  
Abnormal  Pulses  
21. Chronic  aortic  regurgitation  and  aortic  Stenosis-­‐  Pulsus  Bisfierens  
22. Aortic  regurgitation  –  Corrigan’s  pulse  
23. Constructive  pericarditis  –  Pulsus  Paradoxus  
24. Severe  aortic  Stenosis  –  Pulsus  parvus  et  tardus  
25. CHF  –  pulsus  alterans  
 
Cases  
26. Diastolic  blowing  murmur  –  aortic  regurgitation  
th
27. 4  ICS  parasternal  line  –  Tricuspid  regugitation  
28. Ejection  systolic  murmur  transmitted  to  carotid  –  aortic  stenosis  
29. Ejection  systolic  murmur  loud  on  expiration  –  pulmonic  stenosis  
30. Holosystolic  murmur  displaced  to  left  axilla,  diminished  S1  –  Mitral  regurgitation  
 
 
 
1. Important  physical  examination  finding  in  CHF  
+  Hepatojugular  reflux  
2. Mid  diastolic  murmur  heard  over  the  pulmonic  area  in  ASD  is  accompanied  by  what  sound?  
Persistent  splitting  of  S2  
3. Loudness  of  holosystolic  murmur  of  tricuspid  regurgitation  upon  inspiration  
Caravallo’s  Sign  
th
4. Apex  beat  is  displaced  at  the  6  ICS  LAAL,  faint  heart  sound,  (+)  parasternal  heave,  (+)  lift.  
Eccentric  Hypertrophy  
5. 40  year  old  female,  with  apical  beat  that  retracts  during  systole  
RVH  
th
6. SOB,  apex  at  6  ICS  left  midaxillary  line..faint  heart  sound,  alcoholic  and  elicit  drug  use  
Eccentric  
th
7. 40  year  old  with  chronic  hypertension  complained  of  chest  pain.  PE  revealed  apex  beat  @  5  ICS  LMCL  with  a  (+)  
apical  lift.  
Concentric  
8. A  52  year  old  Male,  chronic  hypertension  &  DM2  complained  of  chest  pain  and  SOB.  PE  revealed  (+)  murmur  of  
th
AR  &  MR.  Apex  beat  is  displaced  @  5  ICS  left  anteroaxillary  line  with  (+)  lift.  
Eccentric  
9. Pulmonary  hypertension  
Loud  P2  
10. Atrial  fibrillation    
Soft  S1  
11. BP  220/120  
Loud  S1  
12. RBBB  persistent  
 
13. Sinus  Tachycardia  
Loud  S1  
14. Chronic  Aortic  Regurgitation  
Soft  S1  
15. LBBB  
Paradoxical  splitting  of  S2  
16. Severe  Aortic  Stenosis  
Paradoxical  splitting  of  S2  
  71  
17. Normal  PE  findings  
Split  S2  on  inspiration  
18. Chronic  aortic  regurgitation  and  aortic  stenosis  
Pulsus  bisfiriens  
19. Severe  aortic  stenosis  
 
20. CHF  
Pulsus  Alterans  
 
21. A  27  year  old  female,  worried  about  ECG  finding  of  RBBB.  Auscultation  revealed  presence  of  ejection  systolic  
murmur  which  became  louder  during  forceful  expiration.  
Pulmonic  stenosis  
22. 65  year  old  patient  with  DM,  HPN,  has  a  holosystolic  murmur  displaced  to  the  left  axilla,  has  a  diminished  S1  
Mitral  regurgitation  
23. Machinery-­‐like  murmur  
PDA  
24. Greater  BP  in  the  upper  extremities  compared  to  the  lower  extremities  
Coarctation  of  Aorta  
25. Characteristics  peripheral  pulse  in  CHF  
Pulsus  Alterans  
26. DM  hypertensive  64  year  old  experienced  pain  in  right  calf  muscle  on  walking  for  20  mins  and  relived  by  rest  
Intermittent  Claudication  
27. PDA  
nd
2  Left  ICS  
28. Aortic  Stenosis  
nd
2  Right  ICS  
29. S3  &  S4  
th
5  ICS  LMCL  
30. Mitral  Regurgitation  
 
31. Fixed  splitting  of  S2  
ASD  
32. Holosystolic  murmur  at  the  left  parasternal  area  are  transmitted  to  the  right  sternal  border  
VSD  
33. Sail  Sound  
Ebstein’s  anomaly  
34. BP  elevated  in  the  upper  extremities  markedly  lower  in  the  lower  extremities  
Coarctation  of  Aorta  
35. Machinery-­‐like  murmur  
PDA  
36. Pulmonary  Stenosis  
nd
2  Left  ICS  
 
 
 
 
 
 
  72  
Clinical  Medicine  :  CARDIOVASCULAR  SYSTEM  EXAM  
For  1  to  5,  choices  are:  
a.  concentric       d.  RVH  
b.  eccentric       e.  LVH  
c.  LV  dilataion  
 
1. Not  palpable  in  PE      C  
2. Forceful  apical  beat  but  not  displaced.      A  
3. Markedly  displaced  at  the  left  axillary  line,  hardly  palpable.    C  
4. Displaced  to  axillary  line  5th  or  6th  ICS  anterior  axillary.      B  
5. Palpable  parasternally  but  retract  in  systole.      D  
 
For  6  to  20,  choices  are:  
a.  2nd  LEFT  ICS     d.  4th  ICS  PS  
b.  2nd  RIGHT  ICS   e.  5th  ICS  MCL  
c.  Erb's  point  
 
6. Pulmonic  Stenosis    A  
7. VSD      C  
8.    
9.    
10.    
11. Aortic  valve  regurgitation      A  or  D  
12. Mitral  valve  regurgitation      E  
13. Mitral  Prolapse      E  
14. Aortic  stenosis      B  
15. Mitral  stenosis      E  
16. Loud  S1      E  
17. Opening  snap      A  
18. Loud  P2      A  
19. Hypertrophic  Cardiomyopathy      D  
20. LV  Hypertrophy      E  

 
multiple  choice  ito  na  super  haba,  eto  yung  summary:  
21. Mitral  stenosis  -­‐  diastolic  rumbling  
22. Aortic  stenosis  -­‐  transmitted  to  the  carotid  
23. Aortic  Regurgitation  -­‐  diastolic  rumbling  murmur  
24. VSD  -­‐  pansystolic  

 
For  25  to  30,  choices  are:  
a.  Caravallo's     d.  Gallavardin's  
b.  Kussmaul's     e.  Austin-­‐Flint  
c.  +  hepatojugular     f.  Machinery-­‐like  
 
25. aortic  stenosis      D  
26. tricuspid  regurgitation      A  
27. PDA      F  
28. chronic  aortic  regurgitation      E  
  73  
29. CHF      C  
30. constrictive  pericarditis      B  
 

For  31  to  35,  choices  are:  


a.  loud  P2     d.  opening  snap  
b.  loud  A2     e.  ejection  click  
c.  non-­‐ejection  sound  
31. Mitral  stenosis      D  
32. Mitral  valve  prolapse      C  
33. Constrictive  pericarditis      E  
34. Pulmonary  Hypertension      A  
35. Chronic  uncontrolled  hypertension      B  

 
For  36  to  40,  choices  are:  
a.  Paradoxical  
b.  Wide  
c.  Loud  
d.  Negative  hepatojugular  reflex  
36. Mitral  stenosis    C  (Loud  S2)  
37. Hypertensive  urgency  
38. Pulmonary  hypertension      C  (Loud  S2)  
39. Severe  aortic  stenosis      A  (Paradoxical  splitting  of  S2)  
40. Pulmonary  hypertension      C  (Loud  S2)  
 
For  41  to  45,  choices  are  :  
a.  Pulsus  parvus  et  tardus     d.  negative  hepatojugular  
b.  Corrigan's  pulse       e.  neck  vein  distention  at  60  degrees  
c.  pulsus  bisferiens  
41. normal  PE  finding      D  
42. Pulmonary  edema      E  
43. Aortic  stenosis      A  
44. Combined  aortic  regurgitation  and  aortic  stenosis      C  
45. Chronic  aortic  regurgitation      B  

 
For  46  to  50,  choices  are:  
a.  ejection  systolic  murmur  best  heard  at  
b.  persistent  splitting  
c.  diastolic  murmur  
d.  rumbling  
e.  blowing  
46. Mitral  regurgitation      D  
47. Aortic  stenosis      A  
48. Mitral  stenosis      C  
49. Aortic  stenosis      E  
50. Pulmonic  stenosis      B  
 
 
 
  74  
1. The  anterior  portion  of  the  heart  
a. LV    
b. RV  
c. Aorta  
d. Pulmonary  Artery  
 
2. The  diameter  of  the  apical  beat  
a. 1.5cm  
b. 2.5cm  
c. 2.0cm  
d. .5cm  
 
3. Patient  with  COPD  has  palpable  pulse  at  the  epigastric  and  subxiphoid  region  
a. Abdominal  aorta  
b. RVH  
c. LVD  
d. Biventral  hypertrophy  
 
4. Mitral  stenosis           a.  loud  s1  
5. CAR             b.  soft  s1  
6. Atrial  fibrillation           c.  loud  s2  
7. MSD             d.  ejection  
 
8.               a.  deep  inspiration  
9.               b.  forceful  expiration  
10. MVP             c.  leaning  forward  
d  .LL  decubitous  
 
11. Einsteins  Anomaly         a.  Caravalla’s  
12. Chronic  Tricuspid  Regurgitation       b.  Galavardin’s  
13. Aortic  Stenosis           c.  Snail  sound  
14. Chronic  Severe  Aortic  Regurgitatin       d.  Paradox  pulse  
 
nd
15. Opening  snap  at  rheumatic  mitral  stenosis     a.  2  LICS  
nd
16. Pulmonic  regurgitation         b.  2  RICS  
17. VSD             c.  Parasternal  
18. MVP             d.  Erb’s  
19. Congenital  Pulmonic  Stenosis       e.  Apex  
 
20. Position  of  the  apical  beat  during  physical  exam  
a. upright,  leaning  forward  
b. supine  
c. LLD  
 
 
 
 
 
  75  
21. Normal  location  of  the  apical  beat  
a. Strong  and  forceful  
th
b. Always  at  5  ICP,  L  MCL  
c. Gentle  tap  
d. Strong  during  inspiration  
 
22. Concentric  LV           a.  Persistent  S2  
23. Ischemic  Dilated  Cardiomyopathy       b.  Paradox  S2  
24. Chronic  Severe  Aortic  Stenosis       c.  Physiologic  S2  
25. ASD             d.    
26. Normal  
 
 
 
1. Ventricular  Septal  Defect    
>  HEARD  @  APEX  
2. Very  loud  S1    
>  BEST  HEARD  ON  APEX  
3. Opening  Snap    
>  BEST  HEARD  ON  PULMONIC  AREA  
4. Apical  mid-­‐diastolic  rumbling  murmur    
>  BEST  HEARD  ON  APEX  
5. Chronic  Aortic  Regurgitation  
 >  ERBS  POINT  
6. Acute  Severe  Mitral  Regurgitation  w/  s3  Gallop  
RD
 >  3  LICS  
7. Right–Bundle  Branch  Block    
>  PERSISTENT  /  WIDE  S2  SPLITTING  
8. Left–Bundle  Branch  Block    
>  PARADOXICAL  S2  SPLITTING  
9. Non-­‐Systolic  Click    
>  (MVP)  ERB’S  POINT    
10. Less  Prolapse:  SQUAT  ;  More  Prolapse:  STAND  
11. Ejection  Systolic  Murmur    
(HEARD  LOUDEST  @  BASE)  
12. Atrial  Septal  defect    
ND
>  2  LICS  
13. Concentric  L-­‐ventricular  Hypertrophy  
th
>  5  ICS  LAAL  
14. Restrictive  Hypertrophy  Cardiomyopathy    
>  STANDING  –SQUATTING  POSISTION  
15. Kussmaul  Sign    
>  CAUSE  BY  INABILITY  OF  THE  (R)  HEART  TO  ACCOMODATE  INCREASE  VENOUSE  RETURN  
16. Loud/accentuated  S1    
>  HYPERTENSION  
17. Physiologic  Splitting    
>  2-­‐3  (L)  ICS  
18. Paradoxical  Splitting  of  S2  
>  (L)  BUNDLE  BRANCHING  BLOCK  
 
 
 
 
  76  
 
ABDOMEN  
 
36-­‐40:  Draw  and  label  the  4  quadrants  of  the  abdomen  and  its  landmarks:  
40-­‐50:  Draw  and  label  the  nine  regions  of  the  abdomen  and  its  landmarks:  
 
 
Abdomen  –  Finals  Samplex  
1. A  palpable  left  flank  mass  is  probably  the  left  kidney  if:  
a. You’re  palpating  fingers  can  probe  deep  to  the  medial  and  lateral  borders  (splenomegaly  to)  
b. Preservation  of  normal  tympany  in  LUQ  
c. A  notch  is  palpated  on  medial  border  (splenomegaly  din)  
d. Edge  of  mass  extend  beyond  the  midline  
2. An  extremely  tight  sphincter  tone  on  rectal  examination  is  due  to:  
a. Neuropathy  (seen  in  lax  sphincter  to)   c.  Cerebrovascular  disease  
b. Spinal  cord  lesion       d.  Anxiety  
3. A  35  y/o  male  complains  of  sudden  severe  epigastric  pain  radiating  to  the  back,  duration  noted  after  a  heavy  
meal.  If  this  is  acute  abdominal  pain,  the  following  statement  is/are  correct  
a. Always  mandate  surgical  intervention   c.  Can  be  manage  medially  
b. Duration  of  pain  is  less  than  2  days     d.  Consider  if  the  pain  is  severe  
4. A  21  y/o  male  with  acute  leukemia  have  a  palpable  splenic  notch.  The  traube’s  percussion  will  be:  
a. Hyperresonant         c.  Resonant  
b. Dull             d.  Tympanic  
5. A  60  y/o  diabetic  had  stroke  1  month  ago  with  right  sided  hemiparesis  and  dysarthria..for  sensation  of  food  won’t  
go  down  and  just  stay  on  the  mouth  and  repeatedly  attempts  to  swallow.  This  type  of  dysphagia  is:  
a. Oropharyngeal         c.  Esophageal  dysphagia  
b. Esophageal  web  
6. In  patient  complaining  of  burning  sensation  that  begins  inferiorly  and  radiates  up  to  the  entire  retrosternal  area  
to  the  neck  should  avoid  the  following  except:  
a. NSAIDs       c.  Meperidine     e.  Citrus  food  
b. Coffee       d.  Beta  blockers  
 

______________________________________________________________________________________  
 
Abnormal  contour  –  xymphoid  to  symphisis  pubis  
Portal  HPN:Except  -­‐  SMJ  nodule  
Puddle  sign  –  flicking  sound  becomes  louder,  as  the  stet  goes  farther  
Hypoactive  bowel  –  Ruber  test  
Abdominal  paradox  –  chest  in,  abdomen  expand  upon  inspiration  
Ovarian  cyst  and  ascites  –  hypokalemia,  furosemide,  abdominal  distention  
Difference  of  voluntary  form  involuntary  rigidity  –  Inspire  with  wide  open  mouth  
Carnette’s  sign  –  Abdominal  vs  Intramural  tendencies  
Normal  liver  span  –  4-­‐8  cm  Midsternal,  6-­‐12  RMCL  
Normal  size  of  aorta  –  3  cm  
To  palpate  liver:except  –  Castell  technique  (splenomegaly)  
th
Kidney  punch  –  12  rib,  costovertebral  angle  
Methods  to  assess  Appx:  except  –  Boa’s  sign  
  77  
Sim’s  position  –  Left  knee  flex,  side  of  the  table  
Rectal  exam  in  women  –  mass,  tenderness,  nodules,  lateral  wall,  uterus  
Acute  prostatitis  –  Boddy  and  tender  
Grade  3  –  3-­‐4  cm  protrusion  
Murphy’s  sign  –  Inspiratory  arrest  
Using  Ulnar  surface  of  the  pt  hand  at  midline  –  fluid  wave  
_____________________________________________________________________________________  
 
Alcoholic  portal  hpn  massive  ascites  except:  
a. shifting  dullness         c.  everted  umbilicus  
b. fluid  wave         d.  Typhanitic  
 
Nixon  –  lateral  decubitus  
 
To  confirm  secondary  hpn  
a. Epigastric       c.  R/L  iliac     e.  All  
b. R/L  upper  q       d.  A  and  B  
 
Obese  pt  
a. light  palpate         c.  Reinforced  palpate  
b. deep  palpate         d.  Ballotment  
 
 
 
 
ABDOMEN  
1. Abdominal  contour    →  Xyphoid  to  symphysis  pubis  
2. Portal  Hypertension  →  Except:  SMJ  nodule  
3. Puddle  Sign  →  Flicking  sound  becomes  louder,  as  the  stet  goes  farther  
4. Hypoactive  bowel  →  Hypokalemia,  Furosemide,  Abdominal  distention  
5. Ovarian  cyst  and  ascites  →  Ruler  Test  
6. Differentiate  voluntary  to  involuntary  rigidity  →  Inspire  with  wide  open  mouth  
7. Garnett’s  Sign  →  Abdominal  versus  intramural  tenderness  
8. Normal  liver  span  →  4-­‐8  cm  Midsternal;  6-­‐12  RMCL  
9. Normal  size  of  the  aorta  →  3cm  
10. To  palpate  liver  →  Except:  Castell’s  technique  (for  splenomegaly)  
th
11. Kidney  punch  →  12  rib,  costovertebral  angle  
12. Method  to  assess  AP  (Appendicitis)  →  Except:  Boa’s  Sign  
13. Sim’s  Position  →  Left  knee  flexed  side  of  the  table  
14. Rectal  exam  in  women  →  Mass,  tenderness,  nodules,  lateral  wall,  uterus  
15. Acute  prostatitis  →  Boggy  and  tender  
16. Grade  3  →  3-­‐4cm  protrusion  
17. Murphy’s  sign  →  Inspiratory  arrest  
18. Using  ulnar  surface  of  patient  hand  in  the  midline  →  Fluid  wave  

 
 
  78  
 
ABDOMEN  
1. In  doing  fluid  wave,  the  ulnar  surface  of  the  patient’s  hand  is  pressed  into  the  midline  of  the  abdomen  to  
a. Prevent  movement  of  intestines  
b. Direct  the  movement  of  the  wave  to  the  opposite  side  
c. Block  the  movement  of  the  mesenteric  fat  
d. Clearly  visualize  the  fluid  wave  
 
2. A  60  yo  CHF  for  which  he  was  maintained  on  Furosemide  developed  hypokalemia.  He  has  abdominal  distention  
without  abdominal  pain.  The  auscultatory  findings  will  be  
a. Succusion  splash  
b. Hyperactive  bowel  sounds  
c. Hypoactive  bowel  sounds  
d. Normal  bowel  sounds  
 
3. A  smoker  with  emphysema  was  noted  to  have  palpable  liver  3  finger  breaths  below  the  right  subcostal  margin.  
The  expected  liver  span  is  
a. 4-­‐8  cm  midsternal  
b. 6-­‐12  MCL  
c. 8-­‐14cm  anterior  axillary  line  
d. All  
 
4. All  but  one  are  expected  findings  in  a  46  yo  male  alcoholic  with  portal  hypertension    
(All  are  expected  findings  in  a  46  yo  male  alcoholic  with  portal  hypertension  EXCEPT)  
a. Globular  abdomen  with  everted  umbilicus  
b. Sister  Mary  Joseph  nodules  
c. Spider  angioma  
d. Obliterated  Traube’s  space  
e. (+)  Castells’s  sign  percussion  
 
5. A  19  yo  male  have  severe  attacks  of  bronchial  asthma  was  noted  to  have  abdominal  paradox.  The  abdominal  
respiratory  motion  is  
a. Abdomen  contract  with  expiration  
b. Rocking  motion  of  chest  and  abdomen  
c. Abdomen  expands  while  chest  is  pulled  inward  
d. All  
 
6. To  differentiate  ascites  from  large  ovarian  cyst,  you  must  do  
a. Ballotment  palpation  
b. Deep  palpation  
c. Shifting  dullness  
d. Fluid  wave  
e. Ruler  Test  
 
 
 
 
 
  79  
7. Carnett’s  sign  will  differentiate  (please  double  check  na  lang  po  yung  correct  answer,  thanks!)  
a. Intraabdominal  mass  from  intramural  mass  
b. Rebound  tenderness  from  direct  tenderness  
c. Intraabdominal  tenderness  from  abdominal  tenderness  
d. Subcutaneous  crepitus  from  cutaneous  hyperthesia  
8. A  45  yo  female  complaining  of  RUQ  pain  was  diagnosed  to  have  acute  cholecystitis.  A  positive  Murphy’s  sign  is  
a. Tenderness  on  RUQ  
b. Inspiratory  arrest  
c. (+)  fist  tenderness  on  percussion  of  right  subcostal  margin  
d. Exaggerated  pain  on  gentle  lifting  of  a  fold  of  skin  on  RUQ  
 
9. In  a  22  yo  male  with  RLQ  pain  suspected  to  have  acute  appendicitis,  the  ff  are  positive  abdominal  signs,  EXCEPT  
a. Markle’s  
b. Blumberg’s  
c. Aaron’s  
d. Boa’s    
e. Obturator  
 
10. Positive  puddle’s  sign  is  
a. Distinct  tap  on  your  palpating  hand  
b. The  sound  becomes  louder  while  the  stethoscope  moves  away  from  the  flicking  spot  
c. Dullness  shift  to  the  dependent  side  while  tympanitic  shift  to  the  top  
d. A  visible  movement  as  a  tap  was  done  on  the  other  side  of  the  puddle  
 
11. In  doing  the  kidney  punch,  a  direct  percussion  with  the  fist  should  be  applied  to  
a. Subcostal  margin  
b. Flank  
th
c. 12  rib  and  vertebral  angle  
d. ASIS  
 
12. A  66  yo  male  with  enlarged  prostate  Grade  III  on  rectal  examination,  have  this  amount  of  protrusion  
a. 1-­‐2  cm  
b. >2-­‐3  cm  
c. >3-­‐4  cm  
d. >4  cm    
 
13. A  30  yo  male  complaining  of  dysuria  and  fever  with  pyuria  on  urinalysis.  The  expected  rectal  examination  findings  
if  this  is  acute  bacterial  prostatitis  is/are  
a. Sulcus  is  obliterated  
b. >1  cm  protrusion  
c. Nodular  
d. Boggy  and  tender  
e. All  
 
 
 
 
 
  80  
14. The  ff  statements  is/are  true  with  regards  to  significance  of  rectal  examination  in  female  EXCEPT  
a. Cervix  may  be  palpable  through  anterior  wallt  
b. Uterus  is  never  palpable  on  rectal  examination  
c. Tenderness  of  peritoneal  inflammation  can  be  appreciated  
d. Nodularity  of  peritoneal  metastasis  can  be  felt  
 
15. Sequence  of  abdominal  examination  
ANS.  Inspection,  Auscultation,  Palpation,  Percussion,  Special  Examination  
16. Reference  used  in  determining  abdominal  contour  is  an  imaginary  line  drawn  from  
a. Breast  to  umbilicus  
b. Clavicle  to  the  symphysis  pubis  
c. Rib  margin  to  umbilicus  
d. Xiphoid  or  rib  margin  to  the  pubis  
 
17. To  differentiate  between  abdominal  rigidity  amd  voluntary  muscle  guarding,  examiner  should  do  this  while  doing  
palpation  
a. Distract  the  patient  by  conversation  
b. Ask  the  patient  to  breathe  with  mouth  wide  open  and  feel  relaxation  of  abdomen  during  expiration  
c. Ask  patient  to  raise  his  head  from  supine  position  
d. All  
e. A  and  B  only  
 
18. Technique  to  palpate  a  large  organ  is  freely  movable  as  mass  obscured  by  ascites  
a. Capture  technique  
b. Deep  palpation  
c. Reinforced  palpation  
d. Ballotment  
 
19. Sim’s  position  of  patient  in  rectal  examination  
a. Bent  over  the  examining  table  
b. Lateral  position  lying  on  the  left  side  with  right  hip  and  knee  flexed  with  buttocks  close  to  edge  of  table    
c. Knee-­‐chest  position  
d. Lithotomy  position  
 
20. An  enlarged  prostate  gland  on  rectal  examination  has  the  following  findings  EXCEPT  
a. Firm,  rubbery  consistency  
b. Obliteration  of  sulcus  
c. >1cm  protrusion  into  the  rectum  
d. Lateral  lobes  are  palpable  

 
 
 
 
 
 
 
 
 
  81  
Abdomen  
A.MATCHING  TYPE    

D1.Ovarian  cyst  vs  Ascites  


A2.Obesity  vs.  Ascites  
B3.Intramural  vs.  Intraabdominal  mass  
C4.Peritonitis  vs.  Organomegaly  
E5.-­‐-­‐-­‐-­‐-­‐-­‐-­‐-­‐-­‐-­‐-­‐-­‐-­‐-­‐-­‐-­‐  
A.  Inverted  umbilicus  
B.  Lift  head  while  in  supine  position  
C.  Carnette’s  sign  
D.  Ruler  test  
E.  Traube’s  space  
B.MULTIPLE  CHOICE  
6.Visceral  pain  has  the  following  characteristics  
       A.  More  intense  and  more  localized  
       B.  Patient  move  about  in  an  effort  to  relieve  the  discomfort  
       C.  Aggrevated  by  moving  or  coughing  
       D.  Felt  in  areas  remote  to  diseased  organ  
7.Acute  abdominal  pain,except  
       A.  Approximately  <24hours  
       B.  Sudden  in  onset  and  generalized  
       C.  mandate  surgical  procedure  
         D.  All  of  the  above  
8.Apley  rule  
       A.  The  nearer  from  the  navel  the  pain,the  more  likely  it  will  be  organic  in  origin  
       B.  The  farther  from  the  navel  the  pain,the  least  likely  it  will  be  organic  in  origin  
       C.  The  nearer  from  the  navel  the  pain,the  least  likely  it  will  be  organic  in  origin  
         D.  The  farther  from  the  navel  the  pain,  the  more  likely  it  will  be  organic  in  origin  
 
 
9.Post  myocardial  infarction,  (+)  gag  reflex  
       A.  Pyrosis  
       B.  Globus  pharyngeus  
       C.  Oropharyngeal  dysphagia  
       D.  Esophageal  dysphagia  
10.It  is  the  sensation  of  a  lump  or  tightness  in  the  throat  unrelated  
       A.  Pyrosis  
       B.  Globus  pharyngeus  
       C.  Oropharyngeal  dysphagia    
       D.  Esophageal  dysphagia  
11.Causes  of  heartburn,except  
       A.  Meperidine  
       B.  NSAIDs  
       C.  Coffee  
       D.  Beta  blockers  
       E.  Citrus  food  
12.A  positive  Guiac  test  
       A.  Hematemesis  
       B.  Melena  
       C.  Hematochezia  
       D.  Occult  bleeding  
 
 
 
  82  
13.Type  of  diarrhea  that  has  inhibition  of  sodium  absorption  
       A.Secretory  
       B.  Osmotic  
       C.  Acute  
       D.  Chronic  
14.Etiologic  agents  in  acute  diarrhea,except  
       A.  Vibrio  
       B.  Salmonella  
       C.  Hepatitis  
       D.None  of  the  above  
 
15.Rome  2  classification  of  Chronic  constipation  
       A.  No  bowel  movement  
       B.Straining  
       C.  No  flatus  
       D.  Abnormal  bowel  sounds  
16.A  delay  between  attempting  to  initiate  urination  and  actual  flow  of  urine  
       A.  Overflow  incontinence  
       B.  Stress  incontinence  
       C.Hesitancy  incontinence  
       D.  Reflux  incontinence  
17.Occurs  in  coughing,laughing,exercise,lifting  heavy  objects  
       A.  Overflow  incontinence  
       B.Stress  incontinence  
       C.  Hesitancy  incontinence  
       D.  Reflux  incontinence  
18.Color  of  spun  urine  is  red  
       A.Pigmenturia  
       B.  Hematuria  
       C.  Both  
       D.  Neither  
19.By  indirectly  stimulating  the  chemoreceptor  trigger  zone,vomiting  may  be  initiated  by  followingdrugs  
       A.  Anti-­‐arrhythmias  
       B.  Macrolides  
       C.  Oral  hypoglycemic  agents  
       D.  Opiods  
20.Causes  of  hypoactive  bowel  sounds  
       A.  Diarrhea  
       B.  Early  intestinal  obstruction  
       C.  Hypokalemia  
       D.  Laxatives  
21.A  poitive  cullen  sign  
       A.  Massive  non-­‐traumatic  ecchymoses  
       B.  Acute  hemorrhagic  pancreatitis  
       C.  Noted  on  the  skin  of  the  lower  abdomen  and  flanks  
       D.  Blue-­‐red,blue-­‐purple  or  green-­‐brown  in  color  
22.A  strangulated  hernia  
       A.  Richter’s  hernia  
       B.  Usually  is  almost  always  incarcerated  
       C.  Resembles  pain  of  chronic  peptic  ulcer  
       D.  All  of  the  above  
 
 
 
 
  83  
23.Sounds  produced  by  large  collection  of  air  and  fluid  from  stomach  or  intestine  
       A.  Borborygmi  
       B.  Succussion  splash  
       C.  Peritoneal  friction  rub  
       D.  Fibrillary  hum  
24.Markel’s  sign  is  
       A.  Jar  tenderness  
       B.  Superior  to  rebound  tenderness  as  a  localizing  sign  of  peritoneal  irritation  
       C.  Used  for  ambulatory  patients  with  rigid  abdominall  muscles  
       D.  All  of  the  above  
25.In  a  patient  with  abdominal  aortic  aneurysm  
       A.  There  is  periumbilical  mass  
       B.  There  is  upper  abdominal  mass  
       C.  With  expensile  pulsations  
       D.  All  of  the  above  
26.Normal  abdominal  movement  with  respect  to  respiration  
 
27.Direction  of  blood  flow  in  the  abdomen  
 
 
28.A  palpable  splenic  tail  is  
       A.  Dull  in  percussion  note  
       B.  Suggests  splenic  infection  
       C.  Tympanitic  in  traube’s  space  
       D.  Flat  in  percussion  note  
29.Until  how  many  cm  is  the  liver  edge  palpable  at  the  RUQ  
       A.  1cm  
       B.  2cm  
       C.  3cm  
       D.  4cm  
30.A  normal  prostate  consists  of  the  following,  except  
       A.  Firm  and  rubbery  
       B.  About  2.5cm  in  length  
       C.  Measure  4  x  3  x  2cm  
rd
       D.  3  or  median  lobe  is  composed  of  glandular  tissue  which  is  palpable  
31.An  example  of  referred  pain  
       A.  Pulmanary  tuberculosis  
       B.  Myocardial  infarction  
       C.  Peritonitis  
       D.  Pancreatitis  
32.The  liver  is  palpable  of  about  how  many  cm  during  inspiration  below  the  right  costal  margin  on  the  right  midclavicular  
line?  
       A.  3-­‐4  cm  
       B.  4-­‐8cm  
       C.  6-­‐12cm  
       D.  4-­‐6cm  
33.Bright  red  stool  usually  is  associated  with  
       A.  Lower  GI  bleeding  
       B.  Upper  GI  bleeding  
       C.  Massive  upper  and  lower  GI  bleeding  
       D.  None  of  the  above  
 
 
 
 
  84  
34.Liver  tenderness  is  assed  by  
       A.  Deep  palpation  
       B.  Ballottement  
       C.  Reinforced  palpation  
       D.  All  of  the  above  
35.Ballottement  is  used  to  determine  a  large  organ,  which  is  freely  movable,  or  a  mass  obscured  by  
       A.  Peritonitis  
       B.  Appendicitis  
       C.  Ascites  
       D.  Pancreatitis  
C.DRAW  AND  LABEL  
40-­‐45.  Draw  and  label  the  4  quadrants  of  the  abdomen  and  its  landmarks  
46-­‐50.  Draw  and  label  the  nine  regions  of  the  abdomen  and  its  landmarks  
 
 
 
Clinical  Medicine  Abdomen  
1.  Nocturia  awakening  the  patient  more  than    
a.  Once  (ans)  b.  Twice  c.  3  d.  4  
2.  If  the  liver  is  not  palpable.  To  check  for  liver  tenderness  you  should  elicit?    
A.  Cva  tenderness  B.  Percuss  the  traube's  space.  C.  First  in  the  percussion.  D.  Murphy's  sign  
3.  Signs  ascites  upon  inspection  of  abdomen  
a.  globular  abdomen  b.  bulging  flanks  c.  everted  umbilicus  d.  a&  b  e  all  of  the  above  
4.  Abdominal  hernia  wherein  blood  supply  is  interrupted.    
Answer:  strangulated  
5.  True  about  acute  abdominal  pain.    
Answer:  pain  less  than  24  hours  
6.  Palpation  of  normal  prostate  gland.    
Answer:  firm,  rubbery,  smooth  and  non  tender  
7.  Upon  rectal  examination  the  finger  of  the  examiner  can  reach  up  to    
Answer:  6-­‐10cm  
8.  All  but  one  important  sa  patient  na  may  abdominal  pain  something    
Answer:  “Touch  me  not”  warning  
9.  Manifestation  of  paralytic  ileus.    
Answer:  hypo  or  absent  bowel  sound.  
10.  Renal  bruit  can  be  heard  at.    
Answer:  RUQ/LUQ  
11.  Rulers  test.    
Answer:  large  ovarian  cyst  
12.  Position  of  the  patient  on  abdominal  examination.    
Answer:  supine  with  arms  in  side  or  over  the  chest  with  pillow  under  the  head  and  knee  
13.  Hematemesis    
Answer:  UGIB  
 14.  Abdominal  pain  of  acute  peritonitis?    
aggravated  by  movement  /  coughing  
15.  Pattern  for  abdomen  examination    
ANSWER:  Inspection,  Aus,  palpation,  percussion,  special    examination  
16.  Bowel  sounds  can  be  heard    
ANSWER:  RLQ  (Not  sure  kung  may  ganito)  
17.  Best/preferred  position  for  rectal  examination  of  ambulatory  patients    
ANSWER:  Bend  over  the  examining  table  position  
18.  Present  as  tuft  of  engorged  abdominal  veins  radiating    from  the  umbilicus    
ANSWER:  CAPUT  MEDUSA  
19.  Dysphagia  within  1  second  of  swallowing    
ANSWER:  ORAPHARYNGEAL  DYSPHAGIA  
  85  
 
Abdomen  
th
1. Kidney  Punch-­‐  12  rib  costovertebral  angle  
2. Nodular  mass  (prostate)  except-­‐  3-­‐4cm  node?  
3. Acute  bacterial  prostate-­‐  Tender  and  Boggy  
4. Rectal  tenderness-­‐  except:  Palpation  of  cervix?  
5. Sequence  of  abdominal  examination-­‐  I,  A,  Pal,  Per  
6. Fluid  Wave-­‐  Tapping  1  side  of  abdomen  transmited  to  the  opposite  side  
7. Pain  RLQ  (Appendicitis)  except:  Boa’s  Sign  
8. Puddle  Sign-­‐  +  flicking  is  repeated  while  stetoscope  is  more  farther  away  
9. Abdominal  contour  margin-­‐  Xiphoid  to  Symphysis  Pubis  
10. Sims  Position-­‐lateral  postion  lying  on  his  left  side  with  right  hip  and  knee  somewhat  flexed  
11. Normal  liver  span-­‐6-­‐12cm  MCL  
12. Prostate  enlargement  3-­‐4cm  rectal  protrusion-­‐  Grade  3  
13. Abdominal  Paradox-­‐…..  
14. Carnett  sign-­‐  differentiates  abdominal  tenderness  due  to  inflamed  abdominal  wall  
15. Ascities  vs  Ovarian  Cyst-­‐  Ruler’s  Test  

 
 
 
 
 
Questions  1-­‐10  matching  type  

1.fluid  wave-­‐  NO  VISIBLE  WAVE!  hindi  umaalon  ang  abdomen  accdg  to  macy  

2.  Aaron's  sign-­‐  precordial  pain  

3.  Murphy's  sign-­‐  respiratory  arrest  

4.  Boa's  Sign  

5.  Carnett-­‐patient  raises  his  head  

6.  Castell  

7.  Psoas  

8.  Puddle  

9.  Rovsing  

11.  Which  among  the  ff  applies  to  Apley  Rule?    

A.  Pt  may  give  a  touch  me  not  sign  

B.  Organic  dse  usually  keep  the  pt  eyes  open  

C.  The  farther  from  the  umbilicus,  the  more  organic  

D.  Pt  with  organic  cause  is  generally  not  hungry  

 
  86  
12.  77  y/o  male,  smoker,  developed  dysphagia  that  progressed  to  include  liquids.  What  type  of  disorder?  

A.  Transfer    

B.  Oropharyngeal  

C.  Motility  

D.  Esophageal  (accdg  sa  pinakamamahal  kong  scholar  na  si  larizza,  eto  daw  po  sagot)  

13.  Characteristics  of  Irritable  Bowel  syndrome  

14.  How  much  urine  volume  is  excreted  by  a  person  who  is  oliguric?  <500mL  

15.  Dysphagia  with  psychological  history?  Globus  Pharyngeus  

16.  17,male,  excreting  blood  per  orem,  differentiate  hematemesis  from  hemoptysis?  

A.  Color  of  blood  

B.  Look  for  associated  symptoms  

C.  Do  NGT  

D.  Correlate  clinical  findings  

Ans.  Accdg  again  kay  Lare,  B  sinagot  nya  PERO,  di  sya  sure  kasi  mukhang  ok  din  ung  D.    

P.S  Roi,  ikaw  na  sumagot,  nakalimutan  ko  tanungin  si  Macy  at  Ana  tungkol  dito  

17.  Patient  passing  out  tarry,  black,  foul  smelling  stool,.  Where  is  the  bleeding?  

A.  Upper  GIT  

B.  Lower  GIT  

C.  Obscure  

D.  A  and  B  

E.  All  

Sabi  ni  Macy  E  daw,  sabi  ni  Roi  E  din  daw,  sabi  ko,mkinig  sa  higher  center  ^_^  

18.  Patient  with  heartburn,  you  should  inquire  if  patient  is  taking  all  but  one  in  the  history  taking:  

A.  Coffee  

B.  KCl  tablets  

C.  Theophylline  

D.  Omeprazole    

 
  87  
19.  25  year  old  male  with  dysuria  what  should  you  inquire  about?  

A.  Urethral  Discharge  

B.  Sexual  practices  

C.  Renal  calculi  

D.  A  and  C  

E.  All  

20.  72  yr  old  with  dribbling,  distended  bladder  upon  PE,  What  kind  of  incontinence?  

Ans.  Overflow  incontinence  

21.  Looking  for  secondary  cause  of  HPN,  where  to  look  for  bruit?    

Ans.  Epigastric  and  RUQ/LUQ  

22.  sound  produced  upon  auscultation,  due  to  large  air  and  fluid  production.  Can  be  detected  by  unaided  ear  

Ans.  Succussion  splash  

23.  Sign  of  acute  hemorrhagic  pancreatitis:  Cullen's  

24.  Paradoxical  breathing:  Rocking  motion  (sorry  nakalimutan  na  yung  ibang  choices)  

25.  In  portal  HPN,  direction  of  flow  of  upper  abdominal  veins?  Drains  upward  

26.  Patient  with  history  of  redness  in  the  urine.  Differentiate  hematuria  and  pigmenturia  

A.  Presence  of  blood  clots  

B.  Foamy  urine  

C.  Increase  in  intensity  of  color  when  left  standing  

D.  Reddish  brown  sediments  formed  

 
 
 
 
 
 
 
 
 
 
 
 
 
 
 
 
  88  
ABDOMEN  
 
1.  A  patient  with  ascites  &  obliterated  Traube’s  space  have  a  prominent  abdominal  superficial  veins.  The  expected  
direction  of  blood  flow  on  this  vessel  will  be  
a.  upward  on  upper  abdominal  veins       c.  lateral  at  middle  abdominal  veins  
b.  downward  on  lower  abdominal  veins       d.  all  over  
 
2.  Normally,  the  abdominal  respiratory  motion  is/are  
a.  rocking  motion  of  chest  &  abdomen       d.  all  
b.  abdomen  expand  with  inspiration         e.  b&c        
c.  abdomen  contract  with  expiration    
 
3.  In  umbilical  hernias,  if  the  contents  is  incarcerated  &  blood  supply  is  interrupted,  it  is  called  
a.  reducible           c.  irreducible  
b.  incarcerated           d.  strangulated  
 
4.  A  smoker  with  COPD  on  abdominal  exam  has  a  palpable  liver  3  finger  breathe  below  the  right  subcostal  margin  is  
expected  to  have  a  liver  span  of  
a.  4-­‐6  cm  on  midsternal     c.  8-­‐13cm  on  anterior  axillary  line   e.  a  &  b  only  
b.  6-­‐12  cm  on  MCL     d.  all  
 
5.  A  21  y/o  male  with  acute  leukemia  have  a  palpable  splenic  notch.  The  Traube’s  space  percussion  will  be  
  a.  dull         c.  resonant  
  b.  tympanitic       d.  hyperresonant  
 
6.  To  assess  liver  tenderness  in  a  patient  with  a  liver  abscess  when  the  liver  is  not  palpable,  you  should  do  
  a.  deep  palpation  on  RUQ     c.  reinforced  palpation  
  b.  capture  technique     d.  fist  percussion  of  right  subcostal  margin  
 
7.  In  patient  with  gastric  outlet  obstruction,  the  sound  produced  by  large  collection  of  air  and  fluid  from  stomach,  
detected  by  unaided  ear  is  
  a.  venous  hum       c.  succussion  splash  
  b.  borborygmi       d.  tinkling  bowel  sounds  
 
8.  During  abdominal  examination  of  patient  with  generalized  abdominal  pain,  you  ask  patient  to  stand  on  his  toes  &  
suddenly  drop  his  heels  on  the  floor.  You  are  eliciting  
  a.  Blumberg  sign       c.  Psoas  sign  
  b.  Rovsing’s  sign       d.  Markle’s  sign  
 
9.  A  hypertensive  78  y/o  male  is  being  evaluated  for  abdominal  aortic  aneurysm.  If  this  is  present,  the  expected  findings  
will  be:  
  a.  width  of  aorta  is  >3cm  
  b.  direction  of  aortic  pulsation  is  directly  on  the  palpating  fingers  
  c.  lateral  pulsation  of  aorta  is  noted  
  d.  all  
  e.  a  &  b  only  
 
 
10.  An  80y/o  male,  developed  anuria  &  complaining  of  hypogastric  pain.  Rectal  examination  revealed  grade  4  prostate  
enlargement.  The  expected  abdominal  finding/s  is/are  
a.  palpable  mass  on  hypogastric  area       d.  all  
b.  dullness  on  percussion  of  hypogastric  area       e.  a  &  b  
c.  symmetrically  globular  abdomen  
 
 
  89  
11.  Normal  prostate  gland  on  rectal  examination  has  the  following  findings,  except  
  a.  firm  and  rubbery  in  consistency     c.  nontender  and  movable  
  b.  prominent  median  sulcus     d.  5cm  diameter  with  >1cm  protrusion  
 
12.  On  rectal  examination  the  examining  finger  can  palpate  a  distance  of  
  a.  2-­‐4  cm     b.  4-­‐6  cm   c.  6-­‐10  cm   d.  8-­‐12cm  
 
13.  On  PE  of  the  abdomen,  auscultation  is  done  before  doing  palpation  because:  
  a.  it  will  prevent  ticklishness  of  the  patient  
  b.  palpation  alters  frequency  of  bowel  sounds  
  c.  more  convenient  for  the  examiner  
  d.  it  is  more  preferred  by  the  patient  
 
14.  Reference  used  in  determining  the  abdominal  contour  is  an  imaginary  line  drawn  from:  
  a.  rib  margin  to  the  umbilicus     c.  xiphoid  to  rib  margin  to  the  symphysis  pubis  
  b.  breast  to  the  umbilicus       d.  clavicle  to  the  symphysis  pubis  
 
15.  All  but  one  are  use  to  overcome  ticklishness  of  the  patient  on  palpation  of  the  abdomen,  EXCEPT  
  a.  ask  patient  to  perform  self-­‐palpation  
  b.  talk  to  the  patient  
  c.  place  your  hands  over  patients  finger  and  after  sometime  drift  slowly  your  fingers  onto  the  abdomen  
  d.  use  diaphragm  of  stethoscope  as  palpating  instrument    
 
 
 
 
 
 
 
 
 
 
 
 
 
 
 
 
 
 
 
 
 
 
 
 
 
 
 
 
 
 
 
 
 
  90  
ABDOMEN    
1.  PARIETAL  PAIN  -­‐  More  intense  and  more  precisely  located  -­‐  Aggravated  by  movement  or  coughing    
2.  ACUTE  ABDOMINAL  PAIN  -­‐  <24  hours  -­‐  Not  always  mandate  surgery  -­‐  CAN  be  manage  medically    
3.  APLEY’S  RULE  -­‐  The  farther  from  the  navel/umbilicus,  the  more  likely  it  will  be  organic  in  origin  (only  answer)  -­‐  Other  
choices:  -­‐  Touch  me  not  warning  -­‐  Organic  disease  when  patient  closes/opens  his  eyes    
4.  OROPHARYNGEAL  DYSPHAGIA  -­‐  Food  won’t  go  down  -­‐  Repeatedly  attempts  to  swallow    
5.  ACHALASIA  -­‐  All  are  related  to  disease:  -­‐  With  weight  loss    
6.  All  are  associated  with  HEARTBURN  -­‐  Coffee,  alcohol,  NSAIDS,  nitrates,  theophylline  -­‐  EXCEPT:  -­‐  Colchicine    
7.  HEMATOCHEZIA  -­‐  Bright  red  blood  in  rectum  -­‐  Lower  GI  bleeding  -­‐  Massive  upper  GI  bleeding  (>1L)    
8.  OCCULT  GI  BLEEDING  -­‐  Chronic  anemia  -­‐  No  change  in  color  -­‐  (+)  Guiac’s  test    
9.  BLEEDING  OF  OBSCURE  ORIGIN  -­‐  All  of  the  above    
10.  PEPTIC  ULCER  -­‐  Upper  GI  bleeding  or  massive  bleeding    
11.  AMOEBIC  COLITIS  -­‐  Small  painful  stool  (tenesmus)    
12.  ALLERGY  IN  CRABS  AND  OYSTERS  -­‐  Vibrio  species,  salmonella,  hepatitis  A    
13.  All  are  part  of  ROME  2  CRITERIA  except:  -­‐  Lumpy  or  hard  stool  -­‐  Sensation  of  incomplete  evacuation  -­‐  bowel  action  per  
week    
14.  CONSTIPATION  IN  ELDERLY  -­‐  Decrease  food  intake  -­‐  Weak  abdominal  and  pelvic  muscles  -­‐  Slow  colonic  transit    
15.  HESITANCY  -­‐  Delay  between  attempting  to  initiate  urination  and  actual  flow  of  urine    
16.  CAUSE  OF  PYURIA  -­‐  All  of  the  above    
17.  OLIGURIA  -­‐  <500ml/24hrs    
18.  POLYURIA  -­‐  Increase  osmotic  load  -­‐  Increase  intake  of  fluid  -­‐  ADH  deficiency    
19.  HEMATURIA  -­‐  (+)  RBC  sedimentation    
20.  OVERFLOW  INCONTINENCE  -­‐  A  continuous  dripping  or  dribbling  incontinence    
21.  NIXON’S  TECHNIQUE  -­‐  Percussion  is  initiated  at  lower  level  of  lung  resonance  
22.  REINFORCEMENT  -­‐  Deep  palpation  is  difficult,  obesity  and  deep  seated  pathology    
23.  LIVER  SPAN  -­‐  Accurate  liver  size    
24.  SIM’S  POSITION  -­‐  Patient  lies  on  one  side  with  arms  behind  the  back  and  the  upper  thigh  flexed,  use  in  vaginal  exam    
MATCHING  TYPE    
25.  Ascites  and  obesity  -­‐  Inspection  of  umbilicus    
26.  Ascites  and  ovarian  cyst  -­‐  Ruler  test    
27.  Intramural  and  intraabdominal  -­‐  Patient  raises  his  head  while  in  supine  position    
28.  Abdominal  tenderness  (muscle  strain)  -­‐  Carnet’s  sign    
29.  Splenomegaly  and  hepatomegaly  -­‐  Traube’s  space  
30.  PORTAL  HYPERTENSION  -­‐  Globular  abdomen  (everted)  -­‐  Alcoholic;  liver  cirrhosis  -­‐  (+)  engorge  abdominal  veins  
radiating  from  umbilicus    
31.  JAUNDICE  -­‐  Can  only  be  differentiated  from  carotenemia  in  the  color  of  palm    
32.  CULLEN’S  SIGN  -­‐  Acute  hemorrhagic  pancreatitis    
33.  RESPIRATORY  PARADOX  -­‐  Rocking  motion  of  chest  and  abdomen  
34.  ABDOMINAL  HERNIA  -­‐  Strangulated    
35.  SUCCESSION  SPLASH  -­‐  Large  collection  of  air  and  fluid  from  stomach  and  intestine    
36.  JAR  TENDERNESS  
 
 
 
Personal  note:  Please,  wag  kayo  umasa  dito,  maipasa  niyo  man  ang  clinmed  I,  babagsak  parin  kayo  sa  board  exam  pag  to  
lang  aralin  niyo.  Kung  kasing  tamad  kayo  tulad  ko,  makinig  ka  nalang  ng  mabuti  sa  prof,  3  hours  of  lecture  of  your  24  hrs  is  
not  so  hard  to  give.  
 
-­‐  N    
 
 
 
 
 
 

You might also like